Torts-Negligence, Intentional, and Strict Liability Adaptibar & Midterm Questions

Pataasin ang iyong marka sa homework at exams ngayon gamit ang Quizwiz!

John checked into a Motel and was given a room by the desk clerk. While he slept he was bitten repeatedly by bedbugs. He awoke with red welts and bites all over his arms and legs. It is not unusual to find bedbugs in hotels. Is Motel (who is responsible for clerk's actions) liable for battery? A: Yes, if clerk knew with substantial certainty that the bedbugs would cause a harmful contact. B: Yes, but only if clerk desired to cause a harmful or offensive contact. C: No, because clerk did not intend for the bedbugs to bite John. D: No, because there is no way to know with substantial certainty what bedbugs will do.

A: Yes, if clerk knew with substantial certainty that the bedbugs would cause a harmful contact.

While attending an amusement park's fireworks display, a spectator was struck and injured by a rocket set off as part of the display. The rocket unexpectedly failed to shoot upward but instead followed a trajectory parallel to the ground and struck the spectator. The spectator has sued the amusement park for damages. On which of the following theories is the spectator most likely to be able to obtain a summary judgment as to liability? A: Abnormally dangerous activity. B: Battery. C: Nuisance. D: Strict products liability.

A: Abnormally dangerous activity.

A school bus driver reported to a middle school principal that a student had harassed other children on the bus. The principal informed the student's parents of the bus driver's report and told them that, because of the student's behavior, the student could not ride the bus for the next week and would have to be driven to school by a parent. The following Monday morning, after the bus driver had let the children off the bus in front of the school, but before she could close the door and drive away, the student's father pulled his car directly in front of her bus, blocking the driver's path. Because there was another bus right behind hers, the driver was unable to move her bus. The father got out of his car and strode toward the open door of the bus, screaming at the driver: "You messed with the wrong family! I am going to get you!" Feeling threatened, the bus driver quickly closed the door. The father pounded on the door with enough force to dent it, screaming obscenities at the driver, until a school security guard intervened. If the driver were to sue the father, which cause of action would give her the best chance of recovery? A: Assault. B: Battery. C: Intentional infliction of emotional distress. D: Trespass to chattels.

A: Assault

A patient received anesthesia while giving birth. Upon awakening from the anesthesia, she discovered a severe burn on the inner portion of her right knee. The patient has brought a medical malpractice action in which she has joined all of the physicians and nurses who exercised control over her person, the delivery room, the medical procedures, and the equipment used during the period in which she was unconscious. The defendants have jointly moved for summary judgment. The patient has produced affidavits that establish that the applicable professional standard of care was violated. What would be the patient's best argument against the motion? A: At least one of the defendants had control over whatever agency or instrumentality caused the patient's injury. B: The defendants were acting in concert. C: The patient has produced affidavits that establish that the applicable professional standard of care was violated. D: The patient was in no way responsible for her injury.

A: At least one of the defendants had control over whatever agency or instrumentality caused the patient's injury.

A construction company was digging a trench for a new sewer line in a street in a high-crime neighborhood. During the course of the construction, there had been many thefts of tools and equipment from the construction area. One night, the construction company's employees neglected to place warning lights around the trench. A delivery truck drove into the trench and broke an axle. While the truck driver was looking for a telephone to call a tow truck, thieves broke into the truck and stole $350,000 worth of goods. The delivery company sued the construction company to recover for the $350,000 loss and for the damage to its truck. The construction company has stipulated that it was negligent in failing to place warning lights around the trench and admits liability for damage to the truck, but it denies liability for the loss of the goods. On cross-motions for summary judgment on the claim for the goods, how should the court rule? A: Deny both motions, because there is evidence to support a finding that the construction company should have realized that its negligence could create an opportunity for a third party to commit a crime. B: Grant the construction company's motion, because no one could have foreseen that the failure to place warning lights could result in the loss of a cargo of valuable goods. C: Grant the construction company's motion, because the criminal acts of third persons were a superseding cause of the loss. D: Grant the delivery company's motion, because but for the construction company's actions, the goods would not have been stolen.

A: Deny both motions, because there is evidence to support a finding that the construction company should have realized that its negligence could create an opportunity for a third party to commit a crime.

A longshoreman fell to his death through an open hatch on the deck of a ship. The longshoreman was an employee of a company that had contracted with the ship's owner to load and unload the ship. The fall occurred at night, when loading work was over for the day, and there was no reason for the longshoreman to have been near the hatch. A negligence action was filed against the ship's owner for the death of the longshoreman. In that action, the owner has moved for summary judgment and has provided unrebutted evidence that it is customary for the crews of ships to open the hatches for ventilation after the longshoremen have left the ships. How should the court respond to the motion? A: Deny the motion and submit the case to the jury with instructions that the custom is relevant but not conclusive on the issue of negligence. B: Deny the motion and submit the case to the jury with instructions that the ship's owner should win if the longshoreman was improperly near the hatch. C: Deny the motion, because the probability of serious injury caused by falling down an open hatch clearly outweighs the burden of keeping the hatch closed. D: Grant the motion, because the custom should be considered conclusive on the issue of negligence.

A: Deny the motion and submit the case to the jury with instructions that the custom is relevant but not conclusive on the issue of negligence.

A hotel employed a carefully selected independent contractor to rebuild its swimming pool. The hotel continued to operate while the pool was being rebuilt. The contract between the hotel and the contractor required the contractor to indemnify the hotel for any liability arising from the contractor's negligent acts. A guest of the hotel fell into the excavation, which the contractor had negligently left unguarded. In an action by the guest against the hotel to recover for his injuries, what would be the most likely outcome? A: Liability, because the hotel had a nondelegable duty to the guest to keep a safe premises. B: Liability, because the contract between the hotel and the contractor required the contractor to indemnify the hotel for any liability arising from the contractor's negligent acts. C: No liability, because the contractor was the actively negligent party. D: No liability, because the hotel exercised reasonable care in employing the contractor.

A: Liability, because the hotel had a nondelegable duty to the guest to keep a safe premises.

An employer retained a doctor to evaluate medical records of prospective employees. The doctor informed the employer that an applicant, a prospective employee, suffered from AIDS. The employer informed the applicant of this and declined to hire her. The applicant was shocked by this news and suffered a heart attack as a result. Subsequent tests revealed that the applicant in fact did not have AIDS. The doctor had negligently confused the applicant's file with that of another prospective employee. If the applicant sued the doctor for damages, on which of the following causes of action would the applicant recover? A: Negligent infliction of emotional distress. B: Invasion of privacy. C: Negligent misrepresentation. D: Both invasion of privacy and negligent misrepresentation.

A: Negligent infliction of emotional distress.

A defendant has a small trampoline in his backyard which, as he knows, is commonly used by neighbor children as well as his own. The trampoline is in good condition, is not defective in any way, and normally is surrounded by mats to prevent injury if a user should fall off. Prior to leaving with his family for the day, the defendant leaned the trampoline up against the side of the house and placed the mats in the garage. While the defendant and his family were away, the plaintiff, aged 11, a new boy in the neighborhood, wandered into the defendant's yard and saw the trampoline. The plaintiff had not previously been aware of its presence, but, having frequently used a trampoline before, he decided to set it up, and started to jump. He lost his balance on one jump and took a hard fall on the bare ground, suffering a serious injury that would have been prevented by the mats. An action has been brought against the defendant on the plaintiff's behalf to recover damages for the injuries the plaintiff sustained from his fall. In this jurisdiction, the traditional common-law rules pertaining to contributory negligence have been replaced by a pure comparative negligence rule. In his action against the defendant, will the plaintiff prevail? A: No, because children likely to be attracted by the trampoline would normally realize the risk of using it without mats. B: No, because the plaintiff failed to exercise reasonable care commensurate with his age, intelligence, and experience. C: No, because the plaintiff entered the defendant's yard and used the trampoline without the defendant's permission. D: No, because the plaintiff did not know about the trampoline before entering the defendant's yard and thus was not "lured" onto the premises.

A: No, because children likely to be attracted by the trampoline would normally realize the risk of using it without mats.

A homeowner resides downhill from a metal fabrication facility. She has sued both the owner of the facility and the supplier of a solvent used at the facility. She contends that contaminants, consisting mostly of the solvent, were released into the ground at the facility and have migrated and continue to migrate to her property, contaminating the soil, the groundwater, and her well. She alleges various acts of negligence on the part of the facility owner in causing the release of the contaminants into the ground. She also alleges that employees of the solvent supplier were negligent in frequently spilling some of the solvent onto the ground while filling a rooftop tank at the facility. The solvent supplier has moved for summary judgment, arguing that if there was any contamination, the facility owner and the supplier independently contributed indeterminate amounts to the contamination and that therefore the homeowner cannot show how much damage each has inflicted on her. There is no evidence that the facility owner and the solvent supplier acted in concert. Should the court grant the summary judgment motion? A: No, because concurrent tortfeasors are jointly and severally liable for an indivisible injury. B: No, because the solvent supplier is vicariously liable for damage inflicted by the facility owner. C: Yes, because there is no basis for allocating damages against the solvent supplier. D: Yes, because there is no evidence that the facility owner and the solvent supplier acted in concert.

A: No, because concurrent tortfeasors are jointly and severally liable for an indivisible injury.

A bus passenger was seated next to a woman whom he did not know. The woman stood to exit the bus, leaving a package on the seat. The passenger lightly tapped the woman on the back to get her attention and to inform her that she had forgotten the package. Because the woman had recently had back surgery, the tap was painful and caused her to twist and seriously injure her back. If the woman sues the passenger to recover for the back injury, will she be likely to prevail? A: No, because she is presumed to have consented to the ordinary contacts of daily life. B: No, because she was not put in apprehension by the touching. C: Yes, because the passenger intentionally touched her. D: Yes, because the passenger's intentional touching seriously injured her.

A: No, because she is presumed to have consented to the ordinary contacts of daily life.

The personnel director of an investment company told a job applicant during an interview that the company was worth millions of dollars and that the company's portfolio would triple in the next several months. The applicant was very excited about the company's prospects and accepted an offer to work for the company. Two days later, the applicant read in the newspaper that the investment company had filed for bankruptcy reorganization. As a result of reading this news, the applicant suffered severe emotional distress, but he immediately found another comparable position. Is the applicant likely to prevail in an action for negligent misrepresentation? A: No, because the applicant did not suffer any physical injury or pecuniary loss. B: No, because the personnel director's statement was purely speculative. C: Yes, because the applicant relied on the personnel director's misrepresentations about the investment company. D: Yes, because the personnel director should have foreseen that his misrepresentations would cause the applicant to be distressed.

A: No, because the applicant did not suffer any physical injury or pecuniary loss.

An 11-year-old boy was driving a full-size motorcycle on a private road, where the boy was a trespasser. The motorcycle hit a tire that had fallen off a truck driven by a delivery company employee who was making a delivery to an address on the private road. The boy was injured when his motorcycle went out of control after striking the tire. In a negligence action brought on behalf of the boy against the delivery company, the company contends that the boy was contributorily negligent and that his damages, if any, should be reduced in conformance with the jurisdiction's comparative negligence statute. The boy argues that his conduct should be judged according to the standard of a reasonable child of like age, intelligence, and experience under the circumstances. Is the boy entitled to be judged according to the standard of care that he has argued for? A: No, because the boy was driving a motorcycle. B: No, because the boy was trespassing on the private road. C: Yes, because comparative negligence applies. D: Yes, because the boy was 11 years old at the time.

A: No, because the boy was driving a motorcycle.

A plumbing company hired a worker to work at various construction sites. The worker used his own truck to travel between the company's warehouse and the construction sites, but the company fitted the truck with a rack for carrying plumbing pipes. The company paid the worker for traveling between the warehouse and the construction sites, but not for his drive to and from work. Because the worker was required to haul pipes on his truck while driving between the warehouse and the construction sites, the company asked the worker, before hiring him, whether he had a valid driver's license. The worker represented that he did, although in fact his license had been suspended because he had been convicted of recklessly causing motor vehicle collisions. The company made no effort to verify the worker's representation. While driving to work one morning in his truck, the worker carelessly caused a collision in which a woman was injured. In her subsequent action against the plumbing company, based on a theory of negligent hiring, is the woman likely to prevail? A: No, because the company's duty to use reasonable care in hiring a competent driver extended only to actions taken by the worker in the scope of his employment. B: No, because the worker was an independent contractor. C: Yes, because the company fitted the worker's truck with a pipe rack. D: Yes, because the company had a duty to ensure that its workers had valid driver's licenses.

A: No, because the company's duty to use reasonable care in hiring a competent driver extended only to actions taken by the worker in the scope of his employment.

In a tavern, an intoxicated woman threatened to slash a man with a broken beer bottle. Another customer, who had not been threatened by the woman, forcefully grabbed the woman and locked her in the tavern's storeroom until the police could arrive. In the process, although the customer used reasonable force, the customer badly sprained the woman's wrist. Is the woman likely to recover in an action against the customer? A: No, because the customer's conduct was privileged as a defense of others. B: Yes, based on battery only. C: Yes, based on false imprisonment only. D: Yes, based on both battery and false imprisonment.

A: No, because the customer's conduct was privileged as a defense of others.

A manufacturing plant located near a busy highway uses and stores highly volatile explosives. The owner of the plant has imposed strict safety measures to prevent an explosion at the plant. During an unusually heavy windstorm, a large tile was blown off the roof of the plant and crashed into a passing car, damaging the hood and the windshield. The driver of the car brought a strict liability action against the owner of the plant to recover for the damage to the car. Is the driver likely to prevail? A: No, because the damage to the car did not result from the abnormally dangerous aspect of the plant's activity. B: No, because the severity of the windstorm was unusual. C: Yes, because the plant's activity was abnormally dangerous. D: Yes, because the plant's location near a busy highway was abnormally dangerous.

A: No, because the damage to the car did not result from the abnormally dangerous aspect of the plant's activity.

A shopper was riding on an escalator in a department store when the escalator stopped abruptly. The shopper lost her balance and fell down the escalator steps, sustaining injuries. Although the escalator had been regularly maintained by an independent contractor, the store's obligation to provide safe conditions for its invitees was nondelegable. The shopper has brought an action against the store for damages, and the above facts are the only facts in evidence. The store has moved for a directed verdict. Should the court grant the motion? A: No, because the finder of fact could infer that the escalator malfunction was due to negligence. B: No, because the store is strictly liable for the shopper's injuries. C: Yes, because an independent contractor maintained the escalator. D: Yes, because the shopper has not produced evidence of negligence.

A: No, because the finder of fact could infer that the escalator malfunction was due to negligence.

During a deer-hunting season open to rifle hunters, a hunter saw a deer in the forest. He shot his rifle at the deer, hoping to hit and kill it. Instead, he hit and injured a hiker. The hunter had not realized that the hiker was there. Does the injured hiker have an actionable battery claim against the hunter? A: No, because the hunter did not intend to shoot the hiker. B: No, because the hunter did not make direct physical contact with the hiker. C: Yes, because the bullet from the hunter's rifle made direct physical contact with the hiker. D: Yes, because the hunter intentionally shot the rifle.

A: No, because the hunter did not intend to shoot the hiker.

A man rented a car from a car rental agency. Unbeknownst to the rental agency, the car had a bomb hidden in it at the time of the rental. The bomb exploded an hour later, injuring the man. Immediately prior to renting the car to the man, the rental agency had carefully inspected the car to be sure it was in sound operating condition. The rental agency did not inspect for hidden explosive devices, but such an inspection would have revealed the bomb. There had been no previous incidents of persons hiding bombs in rental cars.In a negligence action by the man against the car rental agency, is the man likely to prevail? A: No, because the rental agency could not have reasonably foreseen the likelihood of someone placing a bomb in the car it was about to rent to the man. B: No, because the rental agency did not hide the bomb in the car. C: Yes, because an inspection for explosive devices would have revealed the bomb. D: Yes, because the bomb made the car abnormally dangerous.

A: No, because the rental agency could not have reasonably foreseen the likelihood of someone placing a bomb in the car it was about to rent to the man

A young woman who attended a rock concert at a nightclub was injured when the band opened its performance with illegal fireworks that ignited foam insulation in the club's ceiling and walls. The young woman sued the radio station that sponsored the performance. The radio station has moved for summary judgment, claiming that it owed no duty to audience members. The evidence has established the following facts: The station advertised its sponsorship on the radio and in print, distributed free tickets to the concert, staffed the event with the station's interns to assist with crowd control, and provided a station disc jockey to serve as master of ceremonies. The master of ceremonies had the authority to stop or delay the performance at any time on the basis of any safety concern. The station knew or should have known that the band routinely used unlicensed, illegal fireworks in its performances. Should the court grant the radio station's motion for summary judgment? A: No, because there is sufficient evidence of knowledge and control on the part of the station to impose on it a duty of care to audience members. B: No, because under respondent superior, the radio station is vicariously liable for the negligent actions of the band. C: Yes, because it is the band and the nightclub owners who owed audience members a duty of care. D: Yes, because the conduct of the band in setting off illegal fireworks was criminal and was a superseding cause as a matter of law.

A: No, because there is sufficient evidence of knowledge and control on the part of the station to impose on it a duty of care to audience members.

An eight-year-old child went to the grocery store with her mother. The child pushed the grocery cart while her mother put items into it. The child's mother remained near the child at all times. Another customer in the store noticed the child pushing the cart in a manner that caused the customer no concern. A short time later, the cart the child was pushing struck the customer in the knee, inflicting serious injury. If the customer brings an action, based on negligence, against the child, the child's best argument in defense would be that... A: The child exercised care commensurate with her age, intelligence, and experience. B: The child is not subject to tort liability. C: The child was subject to parental supervision. D: The customer assumed the risk that the child might hit the customer with the cart.

A: The child exercised care commensurate with her age, intelligence, and experience.

A mother and her six-year-old child were on a walk when the mother stopped to talk with an elderly neighbor. Because the child resented having his mother's attention diverted by the neighbor, the child angrily threw himself against the neighbor and knocked her to the ground. The neighbor suffered a broken wrist as a result of the fall. In an action for battery by the neighbor against the child, what is the strongest argument for liability? A: The child intended to throw himself against the neighbor. B: The child was old enough to appreciate that causing a fall could inflict serious injury. C: The child was old enough to appreciate the riskiness of his conduct. D: The child was not justified in his anger.

A: The child intended to throw himself against the neighbor.

A company set up a website for the advertisement of goods and services offered by individuals, as well as other public notices. One of the notices on the site announced that the furnishings in a home at a specified address were free for the taking. Within a few hours of the posting, all the furnishings had been taken. The notice had been placed by the homeowner's cousin without the homeowner's knowledge. The cousin bore a grudge against the homeowner and had placed the notice while the homeowner was away and had left the door to the home unlocked. In a negligence action brought by the homeowner against the company, what will be the company's strongest defense? A: The company had no duty to the homeowner. B: The cousin's actions were a proximate cause of the homeowner's loss. C: The First Amendment prohibits all tort actions based upon mere speech. D: There is no evidence of careless conduct by the company.

A: The company had no duty to the homeowner.

In a civil action, a plaintiff sued a decedent's estate to recover damages for injuries she suffered in a collision between her car and one driven by the decedent. At trial, the plaintiff introduced undisputed evidence that the decedent's car had swerved across the centerline of the highway into oncoming traffic, where it had collided with the plaintiff's car. The decedent's estate introduced undisputed evidence that, before he swerved across the centerline, the decedent had suffered a fatal heart attack, which he had no reason to foresee, and that, just prior to the heart attack, the decedent had been driving at a reasonable speed and in a reasonable manner. A statute makes it a traffic offense to cross the centerline of a highway. In this case, which party is likely to prevail? A: The decedent's estate, because its rebuttal evidence is undisputed. B: The decedent's estate, because the plaintiff has not established a prima facie case of liability. C: The plaintiff, because the accident was of a type that does not ordinarily happen in the absence of negligence on the actor's part. D: The plaintiff, because the decedent crossed the centerline in violation of the statute.

A: The decedent's estate, because its rebuttal evidence is undisputed.

A man's father died in a hospital. The hospital maintains a morgue with refrigerated drawers a bit larger than the human body. The decedent's body was placed in such a drawer awaiting pickup by a mortician. Before the mortician called for the body, a hospital orderly placed two opaque plastic bags in the drawer with the decedent's body. One bag contained the decedent's personal effects, and the other contained an amputated leg from some other hospital patient. It is stipulated that the hospital was negligent to allow the amputated leg to get into the decedent's drawer. The mortician delivered the two opaque plastic bags to the man, assuming both contained personal effects. The man was shocked when he opened the bag containing the amputated leg. The man sued the hospital to recover for emotional distress. At the trial, the man testified that the experience had been extremely upsetting, that he had had recurring nightmares about it, and that his family and business relationships had been adversely affected for a period of several months. He did not seek medical or psychiatric treatment for his emotional distress. Who should prevail? A: The man, because of the sensitivity people have regarding the care of the bodies of deceased relatives. B: The man, because hospitals are strictly liable for mishandling dead bodies. C: the hospital, because the man did not require medical or psychiatric treatment. D: the hospital, because the man suffered no bodily harm.

A: The man, because of the sensitivity people have regarding the care of the bodies of deceased relatives.

The defendant operates a collection agency. He was trying to collect a valid $400 bill for medical services rendered to the plaintiff by a doctor that was past due. The defendant went to the plaintiff's house and when the plaintiff's mother answered the door, the defendant told her that he was there to collect a bill owed by the plaintiff. The mother told the defendant that because of the plaintiff's illness, the plaintiff had been unemployed for six months, that she was still ill and unable to work, and that she would pay the bill as soon as she could. The defendant, in a loud voice, demanded to see the plaintiff and said that if he did not receive payment immediately, he would file a criminal complaint charging her with fraud. The plaintiff, hearing the conversation, came to the door. The defendant, in a loud voice, repeated his demand for immediate payment and his threat to use criminal process. Assume that the plaintiff did not suffer physical harm as a result of the defendant's conduct, but did suffer severe emotional distress. If the plaintiff asserts a claim against the defendant based on intentional infliction of emotional distress, will the plaintiff prevail? A: Yes, because the plaintiff suffered severe emotional distress as a result of the defendant's conduct. B: No, because the bill for medical services was valid and past due. C: No, because the plaintiff did not suffer physical harm as a result of the defendant's conduct. D: No, because the defendant's conduct created almost no risk of physical harm to the plaintiff.

A: Yes, because the plaintiff suffered severe emotional distress as a result of the defendant's conduct

Patient went to the Doctor to have an irregularly sized mole removed. Patient was very anxious about the procedure, so Doctor gave her a local anesthetic which made her unconscious during the procedure. While Doctor was removing the mole, he noticed a second irregular mole that looked like it should also be removed, so he removed it. Can Patient establish a prima facie case for battery? A: Yes, if the jurisdiction has adopted a single intent standard for battery. B: Yes, if the jurisdiction has adopted a dual intent standard for battery. C: Yes, if the jurisdiction has adopted either a single or dual intent standard for battery. D: Yes, unless the Patient consented to the removal.

A: Yes, if the jurisdiction has adopted a single intent standard for battery.

For five years, a rancher had kept his horse in a ten-acre field enclosed by a six-foot woven wire fence with six inches of barbed wire on top. The gate to the field was latched and could not be opened by an animal. The rancher had never had any trouble with people coming onto his property and bothering the horse, and the horse had never escaped from the field. One day, however, when the rancher went to the field, he found that the gate was open and the horse was gone. Shortly before the rancher's discovery, a driver was driving with due care on a nearby highway when suddenly the rancher's horse darted in front of his car. When the driver attempted to avoid hitting the horse, he lost control of the car, which then crashed into a tree. The driver was injured. The driver sued the rancher to recover damages for his injuries and the rancher moved for summary judgment. If the facts stated above are undisputed, the judge should... A: deny the motion, because pursuant to the doctrine of res ipsa loquitur, a jury could infer that the rancher was negligent. B: deny the motion, because an animal dangerous to highway users escaped from the rancher's property and caused the collision. C: grant the motion, because there is no evidence that the rancher was negligent. D: grant the motion, because the rancher did not knowingly permit the horse to run at large.

A: deny the motion, because pursuant to the doctrine of res ipsa loquitur, a jury could infer that the rancher was negligent. AND C: grant the motion, because there is no evidence that the rancher was negligent.

A supermarket is in a section of town where there are sometimes street fights and where pedestrians are occasionally the victims of pickpockets and muggers. In recognition of the unusual number of robberies in the area, the supermarket posted signs in the store that read: "Warning: There are pickpockets and muggers at work in this part of the city. The supermarket is not responsible for the acts of criminals." Other than posting the signs, the supermarket took no other precautions to prevent criminal activity on the premises. One evening, a customer drove to the supermarket to see about a special on turkeys that the supermarket was advertising. She decided that the turkeys were too large and left the store without purchasing anything. In the parking lot, she was attacked by an unknown man who raped her and then ran away. If the customer sues the supermarket, the result should be for the.... A: plaintiff, because the supermarket failed to take reasonable steps to protect customers against criminal attack in its parking lot. B: plaintiff, because the supermarket is liable for harm to business invitees on its premises. C: defendant, because the warning signs were visible to the customer. D: defendant, because the rapist was the proximate cause of the customer's injuries.

A: plaintiff, because the supermarket failed to take reasonable steps to protect customers against criminal attack in its parking lot.

A professional football player signed a written consent for his team's physician to perform a knee operation. After the athlete was under a general anesthetic, the doctor asked a world famous orthopedic surgeon to perform the operation. The surgeon's skills were superior to the doctor's, and the operation was successful. In an action for battery by the athlete against the surgeon, the athlete will... A: prevail, because the athlete did not agree to allow the surgeon to perform the operation. B: prevail, because the consent form was in writing. C: not prevail, because the surgeon's skills were superior to the doctor's. D: not prevail, because the operation was successful.

A: prevail, because the athlete did not agree to allow the surgeon to perform the operation.

The plaintiff was a passenger in a car that was struck in the rear by a car driven by a student. The collision resulted from the student's negligence in failing to keep a proper lookout. The plaintiff's physician found that the collision had aggravated a mild osteoarthritic condition in her lower back and had brought on similar, but new, symptoms in her neck and upper back. Six months after the first accident, the plaintiff was a passenger in a car that was struck in the rear by a car driven by a doctor. The collision resulted from the doctor's negligence in failing to keep a proper lookout. The plaintiff's physician found that the second collision had caused a general worsening of the plaintiff's condition, marked by a significant restriction of movement and muscle spasms in her back and neck. The physician believes the plaintiff's worsened condition is permanent, and he can find no basis for apportioning responsibility for her present worsened condition between the two automobile collisions. The plaintiff brought an action for damages against the student and the doctor. At the close of the plaintiff's evidence, as outlined above, each of the defendants moved for a directed verdict in his favor on the ground that the plaintiff had failed to produce evidence on which the jury could determine how much damage each defendant had caused. The jurisdiction adheres to the common law rules regarding joint and several liability. The plaintiff's best argument in opposition to the defendants' motions would be that the defendants are jointly and severally liable for the plaintiff's entire harm, because... A: the wrongdoers, rather than their victim, should bear the burden of the impossibility of apportionment. B: the defendants breached a common duty that each of them owed to the plaintiff. C: each of the defendants was the proximate cause in fact of all of the plaintiff's damages. D: the defendants are joint tortfeasors who aggravated the plaintiff's preexisting condition.

A: the wrongdoers, rather than their victim, should bear the burden of the impossibility of apportionment

A defendant's dog ran into the street in front of the defendant's home and began chasing cars. The plaintiff, who was driving a car on the street, swerved to avoid hitting the dog, struck a telephone pole, and was injured. Assume that the defendant knew his dog would often chase cars but refused to restrain it. If the plaintiff asserts a claim against the defendant, will the plaintiff prevail? A: Yes, because the defendant's dog was a cause in fact of the plaintiff's injury. B: Yes, because the defendant knew his dog had a propensity to chase cars and did not restrain it. C: No, because a dog is a domestic animal. D: No, because there is no statute or ordinance making it unlawful for the owner to allow a dog to be unleashed on a public street

B: Yes, because the defendant knew his dog had a propensity to chase cars and did not restrain it.

A hiker sustained a head injury when he was struck by a limb that fell from a tree. At the time of his injury, the hiker was walking through a forest on private property without the property owner's knowledge or permission. It was determined that the limb fell because the tree was infested with termites. In an action by the hiker against the property owner to recover for his head injury, will the hiker be likely to prevail? A: No, because the property owner could not foresee that anyone would be injured. B: No, because the property owner breached no duty to the hiker, who was a trespasser. C: Yes, because the property owner had a duty to prevent the trees on his property from becoming dangerous. D: Yes, because the property owner is liable for hidden dangers on his property.

B: No, because the property owner breached no duty to the hiker, who was a trespasser.

A mother purchased an expensive television from an appliance store for her adult son. Two years after the purchase, a fire started in the son's living room in the middle of the night. The fire department concluded that the fire had started in the television. No other facts are known. The son sued the appliance store for negligence. The store has moved for summary judgment. Should the court grant the store's motion? A: No, because televisions do not catch fire in the absence of negligence. B: No, because the store sold the television. C: Yes, because the son is not in privity with the store. D: Yes, because there is no evidence of negligence on the part of the store.

D: Yes, because there is no evidence of negligence on the part of the store.

A firstborn child was examined as an infant by a doctor who was a specialist in the diagnosis of speech and hearing impairments. Although the doctor should have concluded that the infant was totally deaf due to a hereditary condition, the doctor negligently concluded that the infant's hearing was normal. After the diagnosis, but before they learned that the infant was in fact deaf, the parents conceived a second child who also suffered total deafness due to the hereditary condition. The parents claim that they would not have conceived the second child had they known of the high probability of the hereditary condition. They have sought the advice of their attorney regarding which negligence action against the doctor is most likely to succeed. What sort of action against the doctor should the attorney recommend? A: A medical malpractice action seeking damages on the second child's behalf for expenses related to his deafness, on the ground that the doctor's negligence caused him to be born deaf. B: A wrongful birth action by the parents for expenses they have incurred due to the second child's deafness, on the ground that but for the doctor's negligence, they would not have conceived the second child. C: A wrongful life action by the parents for expenses for the entire period of the second child's life, on the ground that but for the doctor's negligence, the second child would not have been born. D: A wrongful life action on the second child's behalf for expenses for the entire period of his life, on the ground that but for the doctor's negligence, he would not have been born.

B: A wrongful birth action by the parents for expenses they have incurred due to the second child's deafness, on the ground that but for the doctor's negligence, they would not have conceived the second child

A driver negligently ran into a pedestrian who was walking along a road. The pedestrian sustained an injury to his knee, causing it to buckle from time to time. Several months later, the pedestrian sustained an injury to his shoulder when his knee buckled, causing him to fall down a flight of stairs. The pedestrian then brought an action against the driver for the injuries to his knee and shoulder. In his action against the driver, for which of his injuries may the pedestrian recover damages? A: For the injuries to his knee and shoulder, because the driver takes the victim as he finds him. B: For the injuries to his knee and shoulder, if the jury finds that the pedestrian's fall down a flight of stairs was a normal consequence of his original injury. C: For the injury to his knee only, because the injury to the pedestrian's shoulder is separable. D: For the injury to his knee only, if the jury finds that the driver could not have foreseen that his negligent driving would cause the pedestrian to fall down a flight of stairs.

B: For the injuries to his knee and shoulder, if the jury finds that the pedestrian's fall down a flight of stairs was a normal consequence of his original injury.

A woman signed up for a bowling class. Before allowing the woman to bowl, the instructor required her to sign a waiver explicitly stating that she assumed all risk of injuries that she might suffer in connection with the class, including injuries due to negligence or any other fault. After she signed the waiver, the woman was injured when the instructor negligently dropped a bowling ball on the woman's foot. The woman brought a negligence action against the instructor. The instructor has filed a motion for summary judgment based on the waiver. What is the woman's best argument in opposition to the instructor's motion? A: Bowling is an inherently dangerous activity. B: In circumstances like these, it is against public policy to enforce agreements that insulate people from the consequences of their own negligence. C: It was unreasonable to require the woman to sign the waiver before she was allowed to bowl. D: When she signed the form, the woman could not foresee that the instructor would drop a bowling ball on her foot.

B: In circumstances like these, it is against public policy to enforce agreements that insulate people from the consequences of their own negligence.

A patient in a hospital was placed in a wheelchair with his broken leg extended straight out in front of him. As a nurse employed by the hospital was pushing the wheelchair through a set of automatic doors at a normal pace, the doors closed on the patient's foot, injuring it. The nurse attempted to pull the wheelchair back through the doors. This action caused the doors to close more tightly on the patient's foot, injuring it further. The patient sued the hospital, alleging improper maintenance of the doors. The patient has produced no evidence of specific conduct or neglect on the part of the hospital that would have caused the automatic doors to malfunction. The hospital has moved for summary judgment. Should the court grant the hospital's motion? A: No, because a jury could find that there was a latent defect in the doors. B: No, because a jury could find the hospital liable for negligence based on res ipsa loquitur. C: Yes, because proof of an accident, by itself, does not establish that an injured person was a victim of negligence. D: Yes, because the nurse's action was a superseding cause of the injury.

B: No, because a jury could find the hospital liable for negligence based on res ipsa loquitur.

A driver negligently ran over a pedestrian. A bystander witnessed the accident from across the street. The bystander ran to the pedestrian, whom he did not know, and administered first aid, but the pedestrian died in the bystander's arms. The bystander suffered serious emotional distress as a result of his failure to save the pedestrian's life, but he experienced no resulting physical manifestations. The bystander has brought a negligence action against the driver. Is the bystander likely to prevail? A: No, because the bystander assumed the risk. B: No, because the bystander had no familial or other preexisting relationship with the pedestrian. C: Yes, because danger invites rescue. D: Yes, because the bystander was in the zone of danger.

B: No, because the bystander had no familial or other preexisting relationship with the pedestrian.

Toxic materials being transported by truck from a manufacturer's plant to a warehouse leaked from the truck onto the street a few miles from the plant. A driver lost control of his car when he hit the puddle of spilled toxic materials on the street, and he was injured when his car hit a stop sign. In an action for damages by the driver against the manufacturer based on strict liability, is the driver likely to prevail? A: No, because the driver's loss of control was an intervening cause. B: No, because the driver's injury did not result from the toxicity of the materials. C: Yes, because the manufacturer is strictly liable for leaks of its toxic materials. D: Yes, because the leak occurred near the manufacturer's plant.

B: No, because the driver's injury did not result from the toxicity of the materials.

A host pointed an unloaded revolver at her guest, threatening to shoot him. The guest knew that the revolver was not loaded, and that the ammunition for the revolver was stored in a locked basement closet, two stories below where the two were then standing. In an action brought by the guest against the host for assault, will the guest be likely to prevail? A: No, because the host did not intend to shoot her guest. B: No, because the host did not put her guest in apprehension of an imminent contact. C: Yes, because the ammunition was accessible to the host. D: Yes, because the host threatened her guest with a revolver

B: No, because the host did not put her guest in apprehension of an imminent contact.

A Superstore employee regularly stole merchandise from the store. Upon learning of the thefts, the store manager fired the employee, telling him, "If you are not out of this store in ten minutes, I will have the guards throw you out." The employee left at once. If the employee asserts a claim against the manager based on assault will the employee prevail? A: No, because the guards never touched the employee. B: No, because the manager gave the employee ten minutes to leave. C: Yes, if the manager intended to cause the employee severe emotional distress. D: Yes, because the manager threatened the employee with a harmful or offensive bodily contact.

B: No, because the manager gave the employee ten minutes to leave.

A farmer owns a large farm on which he allows his friends to hunt during quail-hunting season. He does not provide his friends with any instructions about gun safety. The neighbor who owns property adjacent to the farm knows of the friends' use of the property during the hunting season. One day during the hunting season, without the farmer's knowledge or permission, the neighbor took a shortcut across the farm to visit an acquaintance. The neighbor was wounded by a shot fired by one of the farmer's friends, who was shooting at quail and carelessly failed to see the neighbor. Traditional rules of landowners' and occupiers' liability apply.In an action by the neighbor against the farmer to recover for the injuries, will the neighbor be likely to prevail? A: No, because the farmer is not responsible for his friends' conduct. B: No, because the neighbor was trespassing. C: Yes, because the careless friend was permitted to hunt without safety training. D: Yes, because the use of firearms is an abnormally dangerous activity

B: No, because the neighbor was trespassing.

A plaintiff suffered from a serious, though not immediately life-threatening impairment of his circulatory system. The plaintiff's cardiologist recommended a cardiac bypass operation and referred the plaintiff to a surgeon. The surgeon did not inform the plaintiff of the 2% risk of death associated with this operation. The surgeon defended his decision not to mention the risk statistics to the plaintiff because the plaintiff "was a worrier and it would significantly lessen his chance of survival to be worried about the nonsurvival rate." The surgeon successfully performed the bypass operation and the plaintiff made a good recovery. However, when the plaintiff learned of the 2% risk of death associated with the operation, he was furious that the surgeon had failed to disclose this information to him, saying that he would have refused the operation if he had known of the risk. If the plaintiff asserts a claim against the surgeon based on negligence, will the plaintiff prevail? A: No, because the surgeon used his best personal judgment in shielding the plaintiff from the risk statistic. B: No, because the operation was successful and the plaintiff suffered no harm. C: Yes, because the plaintiff would have refused the operation had he been informed of the risk. D: Yes, because a patient must be told the risk factors associated with a surgical procedure in order to give informed consent.

B: No, because the operation was successful and the plaintiff suffered no harm.

A driver, returning home from a long work shift at a factory, fell asleep at the wheel and lost control of his car. As a result, his car collided with a police car driven by an officer who was returning to the station after having responded to an emergency. The officer was injured in the accident and later sued the driver in negligence for her injuries. The driver has moved for summary judgment, arguing that the common law firefighters' rule bars the suit. Should the court grant the motion? A: No, because the firefighters' rule does not apply to police officers. B: No, because the police officer's injuries were not related to any special dangers of her job. C: Yes, because the accident would not have occurred but for the emergency. D: Yes, because the police officer was injured on the job.

B: No, because the police officer's injuries were not related to any special dangers of her job.

A bank vice president took substantial kickbacks to approve certain loans that later proved worthless. Upon learning of the kickbacks, the bank's president fired the vice president, telling him, "If you are not out of this bank in 10 minutes, I will have the guards physically throw you out." The vice president left at once. If the vice president asserts a claim against the president based on assault, will the vice president prevail? A: No, because the guards never touched the vice president. B: No, because the president gave the vice president 10 minutes to leave. C: Yes, because the president intended to cause the vice president severe emotional distress. D: Yes, because the president threatened the vice president with a harmful or offensive bodily contact.

B: No, because the president gave the vice president 10 minutes to leave.

During a comprehensive evaluation of an adult patient's psychiatric condition, a psychiatrist failed to diagnose the patient's suicidal state. One day after the misdiagnosis, the patient committed suicide. The patient's father, immediately after having been told of his son's suicide, suffered severe emotional distress, which resulted in a stroke. The patient's father was not present at the patient's appointment with the psychiatrist, nor did he witness the suicide. The father has brought an action against the psychiatrist to recover for his severe emotional distress and the resulting stroke. Is the father likely to prevail? A: No, because the father did not sustain a physical impact. B: No, because the psychiatrist's professional duty did not extend to the harms suffered by the patient's father. C: Yes, because the father was a member of the patient's immediate family. D: Yes, because the psychiatrist reasonably could have foreseen that a misdiagnosis would result in the patient's suicide and the resulting emotional distress of the patient's fathe

B: No, because the psychiatrist's professional duty did not extend to the harms suffered by the patient's father.

A college student was asleep in his bed in a college dormitory when his roommate, in a drunken fury, entered their room intending to attack the student with an ice pick while he slept. Fortunately, the phone rang and awakened the student. The roommate retreated quickly and threw the ice pick under his own bed in the same room. The next day, the student heard from friends about the roommate's murderous plans and later found the ice pick under the roommate's bed. Even though the college expelled his roommate, the student remained extremely upset and afraid to sleep. In a suit against the roommate for assault, will the student prevail? A: The defendant used no more force than he actually believed was necessary to protect himself against death or serious bodily harm. B: The defendant used no more force than he reasonably believed was necessary to protect himself against death or serious bodily harm. C: The defendant, in fact, feared death or serious bodily harm. D: The defendant was justified in retaliating against the plaintiff because the plaintiff struck the first blow.

B: No, because the student was not awake when the roommate entered the room and was unaware until later that the roommate was intending to attack him.

A vintner is the owner of a large vineyard and offers balloon rides to visitors who wish to tour the grounds from the air. During one of the rides, the vintner was forced to make a crash landing on his own property due to high winds. Without the vintner's knowledge or consent, a trespasser had entered the vineyard to camp for a couple of days. The trespasser was injured when he was hit by the basket of the descending balloon. If the trespasser sues the vintner to recover damages for his injuries, will the trespasser prevail? A: No, because there is no evidence that the crash landing was made necessary by the vintner's negligence. B: No, because the vintner was unaware of the trespasser's presence until after the injury had occurred. C: Yes, because even a trespasser may recover for injuries caused by an abnormally dangerous activity. D: Yes, because the accident occurred at a place which the vintner knew was frequented by intruders.

B: No, because the vintner was unaware of the trespasser's presence until after the injury had occurred.

A customer fell and injured himself when he slipped on a banana peel while shopping at a grocery store. The banana peel was fresh and unblemished except for a mark made by the heel of the customer's shoe. In an action brought by the customer against the store, these are the only facts in evidence. Should the trial judge permit the case to go to the jury? A: No, because the customer had an obligation to watch where he stepped. B: No, because there is not a reasonable basis for inferring that the store knew or should have known of the banana peel. C: Yes, because it is more likely than not that the peel came from a banana offered for sale by the grocer. D: Yes, because the store could foresee that a customer might slip on a banana peel.

B: No, because there is not a reasonable basis for inferring that the store knew or should have known of the banana peel.

Borrower owed Lender money. Lender walked toward Borrower, waiving a gun and shouting give me my money. A witness testified Lender raised his hand in the air, making a motion that looked as if Lender planned to hit Borrower with the gun. As Lender raised his hand, the gun discharged and a bullet fired from the gun hit Borrower, causing injury. Lender testified that he did not intend to fire the gun or shoot it at Borrower, and that the gun went off accidentally. Buyer filed suit against Lender, alleging battery. Which of the following statements is true? A: There is direct evidence of Lender's intent to make contact with Borrower. B: There is circumstantial evidence of Lender's intent to harm Borrower. C: There is no direct or circumstantial evidence of Lender's intent to make contact with Borrower. D: There is no evidence of Lender's intent to harm Borrower.

B: There is circumstantial evidence of Lender's intent to harm Borrower.

A complaint filed on behalf of a woman against a nursing home and an ambulance service included the following allegations: The woman, who was 86 years old and unable to speak after suffering a stroke, was picked up from her daughter's house by the ambulance service and taken to the nursing home to stay while her daughter was out of town. When the woman's daughter returned a few days later, the ambulance service picked up the woman from the nursing home and returned her to the daughter's house. The daughter was shocked to discover that the woman had a broken leg; her leg had been uninjured when she left for the nursing home. A physician's report attached to the complaint stated that the woman's leg injury would not have occurred in the absence of negligence. The complaint further alleged that the woman was under the control, successively, of the ambulance service and the nursing home during the time when she must have sustained the injury, and that either the ambulance service or the nursing home must have negligently moved or handled the woman, causing the injury to her leg. Both defendants have argued that the allegations in the complaint are inadequate to support a negligence claim. What is the best response to the defendants' argument? A: Both defendants owed a duty to the woman. B: One of the two defendants probably caused the injury, and the circumstances of the injury are primarily within the knowledge and control of the defendants rather than the woman or her representative. C: The defendants are concurrent tortfeasors, so each is vicariously liable for any tortious act committed by the other. D: There are grounds for the fact-finder to infer that both defendants were negligent.

B: One of the two defendants probably caused the injury, and the circumstances of the injury are primarily within the knowledge and control of the defendants rather than the woman or her representative.

State enacted the following legislation: State shall have no liability for losses resulting from an act or omission by State officer or employee exercising due care in the execution of a valid ordinance. In enacting this statute State has: A: Waived sovereign immunity B: Retained sovereign immunity

B: Retained sovereign immunity

When a jurisdiction adopts a requirement that a plaintiff must prove physical injury to establish the severe emotional distress requirement for proving intentional infliction of emotional distress, it likely has a disproportionate impact (makes it harder to establish a claim) based on a person's: A: Gender. B: Socioeconomic status. C: Political affiliation. D: Ethnicity.

B: Socioeconomic status.

In a plaintiff's action for battery, the evidence established the following: the plaintiff was bad-tempered and, as the defendant knew, carried a gun and used it often; the plaintiff struck the defendant first; during the altercation, the plaintiff repeatedly tried to get to his gun; and the blows inflicted upon the plaintiff by the defendant resulted in the plaintiff being hospitalized. Which finding of fact would be most likely to result in a verdict for the defendant? A: The defendant used no more force than he actually believed was necessary to protect himself against death or serious bodily harm. B: The defendant used no more force than he reasonably believed was necessary to protect himself against death or serious bodily harm. C: The defendant, in fact, feared death or serious bodily harm. D: The defendant was justified in retaliating against the plaintiff because the plaintiff struck the first blow.

B: The defendant used no more force than he reasonably believed was necessary to protect himself against death or serious bodily harm.

While walking on a public sidewalk, a pedestrian was struck by a piece of lumber that fell from the roof of a homeowner's house. The homeowner had hired a repairman to make repairs to his roof, and the lumber fell through due to negligence on the repairman's part. Assume that the homeowner exercised reasonable care in hiring the repairman, that the repairman was an independent contractor, and that public policy made a homeowner's duty to keep the sidewalk safe for pedestrian a nondelegable duty. If the pedestrian brings an action against the homeowner to recover damages for the injury caused to him by the repairman's negligence, will the pedestrian prevail? A: Yes, under the res ipsa loquitur doctrine. B: Yes, because the repairman's act was a breach of a nondelegable duty owed by the homeowner to the pedestrian. C: No, because the repairman was an independent contractor rather than the homeowner's servant. D: No, because the homeowner exercised reasonable care in hiring the repairman to do the repair.

B: Yes, because the repairman's act was a breach of a nondelegable duty owed by the homeowner to the pedestrian.

The day after a seller completed the sale of his house and moved out, one of the slates flew off the roof during a windstorm. The slate struck a pedestrian who was on the public sidewalk. The pedestrian was seriously injured. The roof is old and has lost several slates in ordinary windstorms on other occasions when the seller was present. The pedestrian was also aware that past windstorms had blown slates off the roof. If the pedestrian sues the seller to recover damages for his injuries, will the pedestrian prevail? A: Yes, because the roof was defective when the seller sold the house. B: Yes, because the seller should have been aware of the condition of the roof and should have realized that it was dangerous to persons outside the premises. C: No, because the seller was neither the owner nor the occupier of the house when the pedestrian was injured. D: No, because the pedestrian knew that in the past slates had blown off the roof during windstorms.

B: Yes, because the seller should have been aware of the condition of the roof and should have realized that it was dangerous to persons outside the premises.

A defendant and a group of his friends are fanatical basketball fans who regularly meet at each others' houses to watch basketball games on television. Some of the group are fans of the home team, and others are fans of the rival team. When the group has watched televised games between these two teams, fights sometimes have broken out among the group. Despite this fact, the defendant invited the group to his house to watch a championship game between the home team and the rival team. During the game, the defendant's guests became rowdy and antagonistic. Fearing that they would begin to fight, and that a fight would damage his possessions, the defendant asked his guests to leave. They refused to go and soon began to fight. The defendant called the police, and a police officer was sent to the defendant's home. The officer sustained a broken nose in his efforts to stop the fighting. The officer brought an action against the defendant alleging that the defendant was negligent in inviting the group to his house to watch this championship game. The defendant has moved to dismiss the complaint. The best argument in support of this motion would be that.... A: a rescuer injured while attempting to avert a danger cannot recover damages from the endangered person. B: a police officer is not entitled to a recovery based upon the negligent conduct that created the need for the officer's professional intervention. C: as a matter of law, the defendant's conduct was not the proximate cause of the officer's injury. D: the defendant did not owe the officer a duty to use reasonable care, because the officer was a mere licensee on the defendant's property.

B: a police officer is not entitled to a recovery based upon the negligent conduct that created the need for the officer's professional intervention.

While a driver was taking a leisurely spring drive, he momentarily took his eyes off the road to look at some colorful trees in bloom. As a result, his car swerved a few feet off the roadway, directly toward a pedestrian, who was standing on the shoulder of the road waiting for a chance to cross. When the pedestrian saw the car bearing down on him, he jumped backwards, fell, and injured his knee. The pedestrian sued the driver for damages, and the driver moved for summary judgment. The foregoing facts are undisputed. The driver's motion should be... A: denied, because the record shows that the pedestrian apprehended an imminent, harmful contact, with the driver's car. B: denied, because a jury could find that the driver negligently caused the pedestrian to suffer a legally compensable injury. C: granted, because the proximate cause of the pedestrian's injury was his own voluntary act. D: granted, because it is not unreasonable for a person to be distracted momentarily.

B: denied, because a jury could find that the driver negligently caused the pedestrian to suffer a legally compensable injury.

A construction company was engaged in blasting operations to clear the way for a new road. The company had erected adequate barriers and posted adequate warning signs in the vicinity of the blasting. Although the plaintiff read and understood the signs, he entered the area to walk his dog. As a result of the blasting, the plaintiff was hit by a piece of rock and sustained head injuries. The jurisdiction follows the traditional common law rules governing the defenses of contributory negligence, assumption of risk, and last clear chance. In an action by the plaintiff against the construction company to recover damages for his injuries, the plaintiff will . . . A: not prevail, because the construction company exercised reasonable care to protect the public from harm. B: not prevail, because the plaintiff understood the signs and disregarded the warnings. C: prevail, because the plaintiff was harmed by the construction company's abnormally dangerous activity. D: prevail, because the plaintiff used reasonable care to protect himself from harm

B: not prevail, because the plaintiff understood the signs and disregarded the warnings.

A chemical company manufactured a liquid chemical product known as XRX. Some XRX leaked from a storage tank on the chemical company's property, seeped into the groundwater, flowed to a farmer's adjacent property, and polluted the farmer's well. Several of the farmer's cows drank the polluted well water and died. If the farmer brings an action against the chemical company to recover the value of the cows that died, the farmer will... A: prevail, because a manufacturer is strictly liable for harm caused by its products. B: prevail, because the XRX escaped from the chemical company's premises. C: not prevail, because the farmer is not a foreseeable plaintiff. D: not prevail, because the chemical company was not engaged in an abnormally dangerous activity.

B: prevail, because the XRX escaped from the chemical company's premises.

A builder purchased a large tract of land intending to construct residential housing on it. The builder hired a contractor to build a large in-ground swimming pool on the tract. The contract provided that the contractor would carry out blasting operations that were necessary to create an excavation large enough for the pool. The blasting caused cracks to form in the walls of the plaintiff's home in a nearby residential neighborhood. In the plaintiff's action for damages against the builder, the plaintiff should . . . A: prevail, only if the builder retained the right to direct and control the contractor's construction of the pool. B: prevail, because the blasting that the contractor was hired to perform damaged the plaintiff's home. C: not prevail, if the contractor used reasonable care in conducting the blasting operations. D: not prevail, if the builder used reasonable care to hire a competent contractor.

B: prevail, because the blasting that the contractor was hired to perform damaged the plaintiff's home.

A defendant negligently caused a fire in his house, and the house burned to the ground. As a result, the sun streamed into the plaintiff's yard next door, which previously had been shaded by the defendant's house. The sunshine destroyed some delicate and valuable trees in the plaintiff's yard that could grow only in the shade. The plaintiff has brought a negligence action against the defendant for the loss of the plaintiff's trees. The defendant has moved to dismiss the complaint. The best argument in support of this motion would be that... A: the defendant's negligence was not the active cause of the loss of the plaintiff's trees. B: the defendant's duty to avoid the risks created by a fire did not encompass the risk that sunshine would damage the plaintiff's trees. C: the loss of the trees was not a natural and probable consequence of the defendant's negligence. D: the plaintiff suffered a purely economic loss, which is not compensable in a negligence action.

B: the defendant's duty to avoid the risks created by a fire did not encompass the risk that sunshine would damage the plaintiff's trees.

A driver negligently drove his car into a pedestrian, breaking her leg. The pedestrian's leg was put in a cast, and she used crutches to get around. While shopping at her local supermarket, the pedestrian non-negligently placed one of her crutches on a banana peel that had been negligently left on the floor by the manager of the supermarket's produce department. The pedestrian's crutch slipped on the peel, and she fell to the floor, breaking her arm. Had the pedestrian stepped on the banana peel at a time when she did not have to use crutches, she would have regained her balance. The pedestrian sued the driver and the supermarket for her injuries. The pedestrian will be able to recover from... A: the driver, for her broken leg only. B: the driver, for both of her injuries. C: the supermarket, for both of her injuries. D: the driver, for her broken leg only, and the supermarket, for her broken arm only.

B: the driver, for both of her injuries.

A four-year-old child sustained serious injuries when a playmate pushed him from between two parked cars into the street, where he was struck by a car. The child, by his representative, sued the driver of the car, the playmate's parents, and his own parents. At trial, the child's total damages were determined to be $100,000. The playmate's parents were determined to be 20% at fault because they had failed to adequately supervise her. The driver was found to be 50% at fault. The child's own parents were determined to be 30% at fault for failure to adequately supervise him. The court has adopted the pure comparative negligence doctrine, with joint and several liability, in place of the common law rules relating to plaintiff's fault. In addition, the common law doctrines relating to intra-family liability have been abrogated. What is the maximum amount, if anything, that the child's representative can recover from the driver? A: $30,000. B: $50,000. C: $100,000. D: Nothing.

C: $100,000.

A plaintiff sustained personal injuries in a three-vehicle collision caused by the concurrent negligence of all three drivers. In the plaintiff's action for damages against the other two drivers, the jury apportioned the negligence 30% to the plaintiff, 30% to a truck driver, and 40% to a cab driver. The plaintiff's total damages were $100,000. Assume that the state has retained the common-law rule pertaining to contribution and that the state's comparative negligence statute provides for a system of pure comparative negligence but abolishes joint and several liability. If the plaintiff chooses to pursue the claim against the truck driver alone, she will be entitled to collect at most... A: $70,000 from the truck driver, and then the truck driver will be entitled to collect $40,000 from the cab driver. B: $30,000 from the truck driver, and then the truck driver will be entitled to collect $10,000 from the cab driver. C: $30,000 from the truck driver, and then the truck driver will be entitled to collect nothing from the cab driver. D: nothing from the truck driver, because his percentage of fault is not greater than that of the plaintiff

C: $30,000 from the truck driver, and then the truck driver will be entitled to collect nothing from the cab driver.

A man was admitted to a hospital after complaining of persistent severe headaches. While he was there, hospital staff failed to diagnose his condition, and he was discharged. Two days later, the man died of a massive brain hemorrhage due to a congenital defect in an artery. The man's wife has brought a wrongful death action against the hospital. The wife offers expert testimony that the man would have had a "reasonable chance" (not greater than 50%) of surviving the hemorrhage if he had been given appropriate medical care at the hospital. In what type of jurisdiction would the wife's suit most likely be successful? A: A jurisdiction that applies traditional common law rules concerning burden of proof. B: A jurisdiction that allows recovery based on strict liability. C: A jurisdiction that allows recovery for the loss of the chance of survival. D: A jurisdiction that recognizes loss of spousal consortium.

C: A jurisdiction that allows recovery for the loss of the chance of survival.

A defendant left her car parked on the side of a hill. Two minutes later, the car rolled down the hill and struck and injured the plaintiff. In the plaintiff's negligence action against the defendant, the plaintiff introduced into evidence the facts stated above, which are undisputed. The defendant testified that, when she parked her car, she turned the front wheels into the curb and put on her emergency brakes, which were in good working order. She also introduced evidence that, in the weeks before this incident, juveniles had been tampering with cars in the neighborhood. The jury returned a verdict in favor of the defendant, and the plaintiff properly moved for a judgment notwithstanding the verdict. The plaintiff's motion should be... A: granted, because it is more likely than not that the defendant's negligent conduct was the legal cause of the plaintiff's injuries. B: granted, because the evidence does not support the verdict. C: denied, because, given the defendant's evidence, the jury was not required to draw an inference of negligence from the circumstances of the accident. D: denied, because the defendant was in no better position than the plaintiff to explain the accident.

C: Denied, because, given the defendant's evidence, the jury was not required to draw an inference of negligence from the circumstances of the accident.

Hiker and Friend were on a camping trip. Friend, who knew Hiker had an extreme fear of snakes, put a realistic, but fake snake into Hiker's backpack as a practical joke, thinking it would be funny. When Hiker opened the backpack, she did not realize the snake was fake and suffered a heart attack induced by her fear and panic. Hiker ultimately recovered but must take blood pressure medication for her heart difficulties. If Hiker sues Friend, what result? A: Friend will be liable for battery, if the suit is brought in a dual intent jurisdiction. B: Friend will be liable for battery in a single intent jurisdiction. C: Friend will be liable for intentional infliction of emotional distress. D: Yes, because the woman was in imminent apprehension of a harmful or offensive contact.

C: Friend will be liable for intentional infliction of emotional distress.

A car owner washed her car while it was parked on a public street, in violation of a local ordinance that prohibits the washing of vehicles on public streets during specified hours. The ordinance was enacted only to expedite the flow of automobile traffic. Due to sudden and unexpected cold weather, the car owner's waste water formed a puddle that froze in a crosswalk. A pedestrian slipped on the frozen puddle and broke her leg. The pedestrian sued the car owner to recover for her injury. At trial, the only evidence the pedestrian offered as to negligence was the car owner's admission that she had violated the ordinance. At the conclusion of the evidence, both parties moved for a directed verdict. How should the trial judge proceed? A: Deny both motions and submit the case to the jury, because, on the facts, the jury may infer that the car owner was negligent. B: Deny both motions and submit the case to the jury, because the jury may consider the ordinance violation as evidence that the car owner was negligent. C: Grant the car owner's motion, because the pedestrian has failed to offer adequate evidence that the car owner was negligent. D: Grant the pedestrian's motion, because of the car owner's admitted ordinance violation.

C: Grant the car owner's motion, because the pedestrian has failed to offer adequate evidence that the car owner was negligent.

While approaching an intersection with the red light against him, a motorist suffered a heart attack that rendered him unconscious. The motorist's car struck a child, who was crossing the street with the green light in her favor. Under the state motor vehicle code, it is an offense to drive through a red traffic light. The child sued the motorist to recover for her injuries. At trial it was stipulated that: (1) immediately prior to suffering the heart attack, the motorist had been driving within the speed limit, had seen the red light, and had begun to slow his car; (2) the motorist had no history of heart disease and no warning of this attack; (3) while the motorist was unconscious, his car ran the red light. On cross motions for directed verdicts on the issue of liability at the conclusion of the proofs, the court should.... A: grant the child's motion, because the motorist ran a red light in violation of the motor vehicle code. B: grant the child's motion, because, in the circumstances, reasonable persons would infer that the motorist was negligent. C: grant the motorist's motion, because he had no history of heart disease or warning of the heart attack. D: deny both motions and submit the case to the jury, to determine whether, in the circumstances, the motorist's conduct was that of a reasonably prudent person.

C: Grant the motorist's motion, because he had no history of heart disease or warning of the heart attack.

While driving his pickup truck with a friend riding in the open bed, the driver swerved, throwing his friend to the pavement. The friend sustained severe injuries. The friend had often ridden in the open bed of the truck, and on some of those occasions the driver had swerved to frighten his friend. The friend sued the driver to recover both compensatory damages for his injuries and punitive damages. Which cause of action would NOT permit the friend to recover punitive damages? A: Assault. B: Battery. C: Negligence. D: Recklessness.

C: Negligence

A, intending to throw an empty soda bottle into a trash container, knows that because she has terrible aim, she is almost certain to miss the trash container, and the bottle will strike B who is standing next to the container. The bottle strikes C, who is also standing next to the container, causing injury. Did A intend that consequence and therefore commit a harmful battery on B? A: Yes, because A knew to a substantial certainty the bottle would hit B. B: Yes, because A's intent transfers. C: No, because A did not make the required contact. D: No, because A did not have the required intent

C: No, because A did not make the required contact.

A company designed and built a processing plant for the manufacture of an explosive chemical. An engineer was retained by the company to design a filter system for the processing plant. She prepared an application for a permit to build the plant's filter system and submitted it to the state's Department of Environmental Protection (DEP). As required by DEP regulations, the engineer submitted a blueprint to the DEP with the application for permit. The blueprint showed the entire facility and was signed and sealed by her as a licensed professional engineer. After the project was completed, a portion of the processing plant exploded, injuring the plaintiff. During discovery in an action by the plaintiff against the engineer, it was established that the explosion was caused by a design defect that was unrelated to the filter system designed by the engineer. However, the defect was present in the blueprint signed by the engineer. In that action, will the plaintiff prevail? A: Yes, because the engineer signed, sealed, and submitted a blueprint that showed the design defect. B: Yes, because all of the plant's designers are jointly and severably liable for the defect. C: No, because the engineer owed no duty to the plaintiff to prevent the particular risk of harm. D: No, because the engineer was an independent contractor.

C: No, because the engineer owed no duty to the plaintiff to prevent the particular risk of harm.

A hiker, although acting with reasonable care, fell while attempting to climb a mountain and lay unconscious and critically injured on a ledge that was difficult to reach. The plaintiff, an experienced mountain climber, was himself seriously injured while trying to rescue the hiker. The plaintiff's rescue attempt failed, and the hiker died of his injuries before he could be reached. The plaintiff brought an action against the hiker's estate for compensation for his injuries. In this jurisdiction, the traditional common-law rules relating to contributory negligence and assumption of risk remain in effect. Will the plaintiff prevail in his action against the hiker's estate? A: Yes, because his rescue attempt was reasonable. B: Yes, because the law should not discourage attempts to assist persons in helpless peril. C: No, because the hiker's peril did not arise from his own failure to exercise reasonable care. D: No, because the plaintiff's rescue attempt failed and therefore did not benefit the hiker.

C: No, because the hiker's peril did not arise from his own failure to exercise reasonable care.

A driver was driving his car near a homeowner's house when the homeowner's child darted into the street in front of the driver's car. As the driver swerved and braked his car to avoid hitting the child, the car skidded up into the homeowner's driveway and stopped just short of the homeowner, who was standing in the driveway and had witnessed the entire incident. The homeowner suffered from serious emotional distress from witnessing the danger to his child and to himself. Neither the homeowner nor his property was physically harmed. If the homeowner asserts a claim for damages against the driver but is unable to establish that the driver was negligent, will the homeowner still be able to prevail? A: Yes, because the driver's entry onto the homeowner's land was unauthorized. B: Yes, because the homeowner suffered serious emotional distress by witnessing the danger to his child and to himself. C: No, because the homeowner failed to show that the driver was negligent. D: No, because the homeowner's child was not exercising reasonable care.

C: No, because the homeowner failed to show that the driver was negligent.

A child was bitten by a dog while playing in a fenced-in common area of an apartment complex owned by a landlord. The child was the guest of a tenant living in the complex, and the dog was owned by another tenant. The owner of the dog knew that the dog had a propensity to bite, but the landlord did not have any notice of the dog's vicious propensities. In an action by the child against the landlord, will the child be likely to prevail? A: Yes, because in these circumstances a landlord is strictly liable. B: Yes, because a landlord's duty to protect a tenant's guests from dangerous conditions is nondelegable. C: No, because the landlord did not have notice of the dog's vicious propensities. D: No, because a landlord owes no duty to a tenant's guests.

C: No, because the landlord did not have notice of the dog's vicious propensities.

A passenger departed on an ocean liner knowing that it would be a rough voyage due to predicted storms. The ocean liner was not equipped with the type of lifeboats required by the applicable statute. The passenger was swept overboard and drowned in a storm so heavy that even a lifeboat that conformed to the statute could not have been launched. In an action against the operator of the ocean liner brought by the passenger's representative, will the passenger's representative prevail? A: Yes, because the ocean liner was not equipped with the statutorily required lifeboats. B: Yes, because in these circumstances common carriers are strictly liable. C: No, because the storm was so severe that it would have been impossible to launch a statutorily required lifeboat. D: No, because the passenger assumed the risk by boarding the ocean liner knowing that it would be a rough voyage

C: No, because the storm was so severe that it would have been impossible to launch a statutorily required lifeboat.

A real estate developer was trying to purchase land on which he intended to build a large commercial development. An elderly widow had rejected all of the developer's offers to buy her ancestral home, where she had lived all her life and which was located in the middle of the developer's planned development. Finally, the developer offered her $250,000. He told her that if she rejected it, state law authorized him to have her property condemned. He subsequently parked a bulldozer in front of her house. The widow then consulted her nephew, a law student, who researched the question and advised her that the developer had no power of condemnation under state law. The widow had been badly frightened by the developer's threat, and was outraged when she learned that the developer had lied to her. If the widow sues the developer for damages for emotional distress, will she prevail? A: Yes, because the developer's action was extreme and outrageous. B: Yes, because the widow was frightened and outraged. C: No, because the widow did not suffer emotional distress that was severe. D: No, because it was not the developer's purpose to cause emotional distress.

C: No, because the widow did not suffer emotional distress that was severe.

A mother rushed her eight-year-old daughter to the emergency room at a local hospital after the child fell off her bicycle and hit her head on a sharp rock. The wound caused by the fall was extensive and bloody. The mother was permitted to remain in the treatment room, and held the child's hand while the emergency room physician cleaned and sutured the wound. During the procedure, the mother said that she was feeling faint and stood up to leave the room. While leaving the room, the mother fainted and, in falling, struck her head on a metal fixture that protruded from the emergency room wall. She sustained a serious injury as a consequence. If the mother sues the hospital to recover damages for her injury, will she prevail? A: Yes, because the mother was a public invitee of the hospital's. B: Yes, because the fixture was not an obvious, commonly used, and essential part of the hospital's equipment. C: No, because there is no evidence that the hospital's personnel failed to take reasonable steps to anticipate and prevent the mother's injury. D: No, because the hospital's personnel owed the mother no affirmative duty of care.

C: No, because there is no evidence that the hospital's personnel failed to take reasonable steps to anticipate and prevent the mother's injury.

The plaintiff, a jockey, was seriously injured in a race when another jockey, the defendant, cut too sharply in front of her without adequate clearance. The two horses collided, causing the plaintiff to fall to the ground, sustaining injury. The State Racetrack Commission ruled that, by cutting in too sharply, the defendant committed a foul in violation of racetrack rules requiring adequate clearance for crossing lanes. The plaintiff has brought an action against the defendant for damages in which one count is based on battery. Will the plaintiff prevail on the battery claim? A: Yes, if the defendant was reckless in cutting across in front of the plaintiff's horse. B: Yes, because the State Racetrack Commission determined that the defendant committed a foul in violation of rules applicable to racing. C: No, unless the defendant intended to cause impermissible contact between the two horses or apprehension of such contact by the plaintiff. D: No, because the plaintiff assumed the risk of accidental injury inherent in riding as a jockey in a horse race.

C: No, unless the defendant intended to cause impermissible contact between the two horses or apprehension of such contact by the plaintiff.

A patient had been under the care of a cardiologist for three years prior to submitting to an elective operation that was performed by a surgeon. Two days thereafter, the patient suffered a stroke, resulting in a coma, caused by a blood clot which formed after the operation. When it appeared that she had entered a permanent vegetative state, with no hope of recovery, the artificial life-support system that had been provided was withdrawn, and she died a few hours later. The withdrawal of artificial life support had been requested by her family, and duly approved by a court. The surgeon was not involved in that decision, or in its execution. The administrator of the patient's estate thereafter filed a wrongful death action against the surgeon, claiming that the surgeon was negligent in having failed to consult a cardiologist prior to the operation. At the trial the plaintiff offered evidence that accepted medical practice would require examination of the patient by a cardiologist prior to the type of operation that the surgeon performed. In this action, the plaintiff should.... A: prevail, because the surgeon was negligent in failing to have the patient examined by a cardiologist prior to the operation. B: prevail, because the blood clot that caused the patient's death was caused by the operation which the surgeon performed. C: not prevail, because there is no evidence that a cardiologist would have provided advice that would have changed the outcome if one had examined the patient before the operation. D: not prevail, because the surgeon had nothing to do with the withdrawal of artificial life support, which was the cause of the patient's death.

C: Not prevail, because there is no evidence that a cardiologist would have provided advice that would have changed the outcome if one had examined the patient before the operation.

Which of the following, if true, would provide the best support for plaintiff's claim that her signed consent form for a tubal ligation is invalid: A: The consent form was in a language that plaintiff did not read or speak. B: Plaintiff did not understand what she was signing. C: Plaintiff was told by her doctor that if she did not sign the form her baby might die. D: Plaintiff was in labor when she signed the document.

C: Plaintiff was told by her doctor that if she did not sign the form her baby might die.

In a trial by jury, a restaurant owner proved that a power company's negligent maintenance of a transformer caused a fire that destroyed his restaurant. The jury returned a verdict for the owner in the amount of $450,000 for property loss and $500,000 for emotional distress. The trial judge entered judgment in those amounts. The power company appealed the part of the judgment awarding $500,000 for emotional distress. On appeal, the judgment should be... A: affirmed, because the power company negligently caused the owner's emotional distress. B: affirmed, because harm arising from emotional distress is as real as harm caused by physical impact. C: reversed, because the law does not recognize a claim for emotional distress incident to negligently caused property loss. D: reversed, because the owner suffered physical harm as a consequence of the emotional distress caused by his property loss.

C: Reversed, because the law does not recognize a claim for emotional distress incident to negligently caused property loss.

A bright 12-year-old child attended a day-care center after school. The center was located near a man-made duck pond on the property of a corporation. During the winter, the pond was used for ice-skating when conditions were suitable. At a time when the pond was obviously only partially frozen, the child sneaked away from the center's property and walked out onto the ice over the pond. The ice gave way, and the child fell into the cold water. He suffered shock and would have drowned had he not been rescued by a passerby. At the time of the incident, the pond was clearly marked with numerous signs that stated, "THIN ICE—KEEP OFF." When the child sneaked away from the day-care center, the center was staffed with a reasonable number of qualified employees, and the employees were exercising reasonable care to ensure that the children in their charge did not leave the premises. There had not been a previous instance of a child coming onto the corporation's property from the day-care center. The jurisdiction follows a rule of pure comparative negligence. In a suit brought on the child's behalf against the day-care center and based only on the facts above, who is likely to prevail? A: The child, because he left the center while he was under the center's care. B: The child, because the day-care center is located near a pond. C: The day-care center, because it was not negligent. D: The day-care center, because the child was a trespasser.

C: The day-care center, because it was not negligent.

Under the Federal Tort Claims Act, with certain exceptions not relevant here, the federal government is liable only for negligence. A federally owned and operated nuclear reactor emitted substantial quantities of radioactive matter that settled on a nearby dairy farm, killing the dairy herd and contaminating the soil. At the trial of an action brought against the federal government by the farm's owner, the trier of fact found the following: (1) the nuclear plant had a sound design, but a valve made by an engineering company had malfunctioned and allowed the radioactive matter to escape; (2) the engineering company was universally regarded as a quality manufacturer of components for nuclear plants; and (3) there was no way the federal government could have anticipated or prevented the emission of the radioactive matter. If there is no other applicable statute, for which party should the court enter judgment? A: The farm owner, on the ground that the doctrine of res ipsa loquitur applies. B: The farm owner, on the ground that one who allows dangerous material to escape to the property of another is liable for the damage done. C: The government, on the ground that a case under the Federal Tort Claims Act has not been proved. D: The government, on the ground that the engineering company is the proximate cause of the farm owner's damage.

C: The government, on the ground that a case under the Federal Tort Claims Act has not been proved.

A recently installed elevator suddenly started free-falling down the elevator shaft while carrying passengers. Frightened, a passenger pried the inside doors open and impulsively stuck his arm through them to try to stop the fall. As a result, his arm was broken. The elevator eventually stopped without causing further injuries. In a negligence action brought by the injured passenger against the company that installed and maintained the elevator, the injured passenger has asked the trial judge to instruct the jury that it may find the company negligent on a theory of res ipsa loquitur. In response, the company has argued that the passenger's conduct caused his injuries. How should the judge rule? A: The judge should deny the passenger's request, because it is possible that the company was not negligent. B: The judge should deny the passenger's request, because the jury could find that the conduct of the passenger contributed to his injuries. C: The judge should grant the passenger's request but should also instruct the jurors to consider any carelessness of the passenger in awarding damages if they find the company liable. D: The judge should grant the passenger's request, because the passenger acted reasonably considering the stress of the situation.

C: The judge should grant the passenger's request but should also instruct the jurors to consider any carelessness of the passenger in awarding damages if they find the company liable.

A mining company that operated a copper mine in a remote location kept dynamite in a storage facility at the mine. The storage facility was designed and operated in conformity with state-of-the-art safety standards. In the jurisdiction, the storage of dynamite is deemed an abnormally dangerous activity. Dynamite that was stored in the mining company's storage facility and that had been manufactured by an explosives manufacturer exploded due to an unknown cause. The explosion injured a state employee who was at the mine performing a safety audit. The employee brought an action in strict liability against the mining company. What would be the mining company's best defense? A: The mine was in a remote location. B: The mining company did not manufacture the dynamite. C: The state employee assumed the risk of injury inherent in the job. D: The storage facility conformed to state-of-the-art safety standards

C: The state employee assumed the risk of injury inherent in the job.

In an action by a man against a pharmacy, the man offered only the following evidence: The man took a clearly written prescription to a pharmacy. The pharmacy's employee filled the prescription by providing pills with 30 milligrams of the active ingredient instead of 20 milligrams, as was prescribed. Shortly after taking the pills as directed, the man, who had no previous history of heart problems, suffered a heart attack. Overdoses of the active ingredient had previously been associated with heart problems. Does the man have a valid claim against the pharmacy? A: No, because pharmacies are not strictly liable for injuries caused by incorrectly filled prescriptions. B: No, because the man offered no specific proof as to the pharmacy's negligence. C: Yes, because a jury could reasonably conclude that the man would not have suffered a heart attack had the pharmacy provided the correct dosage. D: Yes, because by providing the 30-milligram pills rather than the 20-milligram pills, the pharmacy sold the man a defective product.

C: Yes, because a jury could reasonably conclude that the man would not have suffered a heart attack had the pharmacy provided the correct dosage.

A customer bought a can of corn at a grocery store. While eating the corn later that evening, the customer was injured by a small piece of glass in the corn. The customer sued the canning company that had processed and canned the corn. At trial, the customer presented evidence that neither the customer nor any third party had done anything after the can of corn was opened that would account for the presence of the glass. Without any other evidence, is the customer likely to prevail? A: No, because it is possible that someone tampered with the can before the customer bought it. B: No, because the customer has not shown any direct evidence that the canning company acted negligently. C: Yes, because a jury may reasonably infer that the canning company acted negligently. D: Yes, because the grocery store could not have discovered the piece of glass by reasonable inspection

C: Yes, because a jury may reasonably infer that the canning company acted negligently.

A man's car sustained moderate damage in a collision with a car driven by a woman. The accident was caused solely by the woman's negligence. The man's car was still drivable after the accident. Examining the car the next morning, the man could see that a rear fender had to be replaced. He also noticed that gasoline had dripped onto the garage floor. The collision had caused a small leak in the gasoline tank. The man then took the car to a mechanic, who owns and operates a body shop, and arranged with the mechanic to repair the damage. During their discussion the man neglected to mention the gasoline leakage. Thereafter, while the mechanic was loosening some of the damaged material with a hammer, he caused a spark, igniting vapor and gasoline that had leaked from the fuel tank. The mechanic was severely burned. The mechanic has brought an action to recover damages against the man and woman. The jurisdiction has adopted a pure comparative negligence rule in place of the traditional common law rule of contributory negligence. The jury found that while a reasonable person in the man's position would have warned the mechanic about the gasoline leak, the man had no actual knowledge of the risk that the gasoline leak presented. In this action, will the mechanic obtain a judgment against the man? A: No, because it was the mechanic's job to inspect the vehicle and repair whatever needed repair. B: No, because the man was not aware of the risk that the gasoline leak presented. C: Yes, because a reasonable person in the man's position would have warned the mechanic about the gasoline leak. D: Yes, because the car was unreasonably dangerous when the man delivered it to the mechanic.

C: Yes, because a reasonable person in the man's position would have warned the mechanic about the gasoline leak.

A man and a woman were competing in an illegal drag race. Both of them were driving over the speed limit but were otherwise driving very carefully. However, when a tire on the woman's car suddenly blew out, she lost control of her car and crashed, injuring a pedestrian. The pedestrian later sued the man, because the woman had no insurance or assets. Will the pedestrian be likely to prevail in that action? A: No, because the man did not cause the injury. B: No, because the man was driving very carefully. C: Yes, because the man and the woman were acting in concert in a dangerous activity. D: Yes, because the man was exceeding the speed limit.

C: Yes, because the man and the woman were acting in concert in a dangerous activity.

A doctor ordered chest X-rays for a patient who smoked cigarettes. After the consulting radiologist told the doctor that the X-rays looked normal, the doctor told the patient that he was in good health. In fact, the radiologist had missed signs of cancer on the X-rays that a trained radiologist, acting competently, would have detected. After another X-ray of the patient's chest, performed one year later, showed advanced lung cancer, the doctor discovered that the radiologist had misinterpreted the patient's earlier X-rays. The patient died within four months of the later X-ray, because by then his cancer had become untreatable. In a wrongful death suit against the radiologist based on only the facts set out above, a jury found the radiologist negligent and awarded $3 million in compensatory damages and $21 million in punitive damages. Is the radiologist likely to have the punitive damages award vacated on appeal? A: No, because a 7 to 1 ratio of punitive to compensatory damages is constitutionally permissible. B: No, because an award of punitive damages is appropriate for medical malpractice that results in death or serious injury. C: Yes, because punitive damages awards are not authorized unless there is proof of willful or wanton misconduct on the defendant's part. D: Yes, because the patient smoked cigarettes and therefore was contributorily negligent.

C: Yes, because punitive damages awards are not authorized unless there is proof of willful or wanton misconduct on the defendant's part.

A fumigation company was hired to eliminate pests in one of two buildings in a condominium complex that shared a common wall. The owners of the complex told the fumigation company that the common wall separating the infested building from the uninfested building was an impenetrable fire wall. The fumigation company did its own thorough inspection and determined that the buildings were indeed completely separated by the wall. Residents of the condominium units in the building that was to be sprayed were told to evacuate, but the residents of the uninfested building were told that they could remain while the other building was treated. During and shortly after the fumigation, in which a highly toxic chemical was used, many residents of the uninfested building became sick. It was determined that their illnesses were caused by the fumigation chemical. In fact, there was a hole in the fire wall separating the two buildings, but because it could only be observed from a specific position in the crawl space underneath the floor of the uninfested building, it had not been discovered by either the fumigation company or any previous building inspector. Are the residents of the uninfested building likely to prevail in a tort action against the fumigation company? A: No, because the condominium complex owners were responsible for accurately conveying the condition of their buildings. B: No, because the fumigation company exercised a high level of care. C: Yes, because the fumigation company can be held strictly liable for its activity. D: Yes, because the fumigation company put a dangerous product into the stream of commerce.

C: Yes, because the fumigation company can be held strictly liable for its activity.

A 14-year-old girl of low intelligence received her parents' permission to drive their car. She had had very little experience driving a car and did not have a driver's license. Although she did the best she could, she lost control of the car and hit a pedestrian. The pedestrian has brought a negligence action against the girl. Is the pedestrian likely to prevail? A: No, because only the girl's parents are subject to liability. B: No, because the girl was acting reasonably for a 14-year-old of low intelligence and little driving experience. C: Yes, because the girl was engaging in an adult activity. D: Yes, because the girl was not old enough to obtain a driver's license

C: Yes, because the girl was engaging in an adult activity.

A plaintiff's three-year-old daughter was killed in an automobile accident. At the plaintiff's direction, the child's body was taken to a mausoleum for interment. Normally, the mausoleum's vaults are permanently sealed with marble plates secured by "tamper-proof" screws. After the child's body was placed in a mausoleum, however, only a fiberglass panel secured by caulking compound covered her vault. About a month later, the child's body was discovered in a cemetery located near the mausoleum. It had apparently been left there by vandals who had taken it from the mausoleum. As a result of this experience, the plaintiff suffered great emotional distress.If the plaintiff sues the mausoleum for the damages arising from her emotional distress, will she prevail? A: No, because the plaintiff experienced no threat to her own safety. B: No, because the mausoleum's behavior was not extreme and outrageous. C: Yes, because the mausoleum failed to use reasonable care to safeguard the body. D: No, because the plaintiff suffered no physical harm as a consequence of her emotional distress.

C: Yes, because the mausoleum failed to use reasonable care to safeguard the body.

A patient who had suffered a severe fracture of her leg was treated by an orthopedist, who set the patient's leg and put it in a cast. When the leg continued to bother the patient six months later, she consulted a second orthopedist in the same town. The second orthopedist surgically inserted a pin to facilitate healing. The patient brought a malpractice action against the first orthopedist, claiming that he should have surgically inserted a pin at the time of initial treatment. The only evidence that the patient offered in support of her malpractice claim was the testimony of the second orthopedist, as follows: In response to the question "Would you have inserted a pin initially?" the second orthopedist testified, "I personally would not have been satisfied that the leg would heal properly without a pin." At the close of the patient's evidence, the first orthopedist moved for judgment as a matter of law. Should the motion be granted? A: No, because the patient has introduced evidence that the first orthopedist failed to give the care that the second orthopedist would have provided. B: No, because the second orthopedist practices in the same town and field of specialty as the first orthopedist. C: Yes, because the patient has failed to introduce evidence that the first orthopedist's care fell below the professional standard of care. D: Yes, because the second orthopedist also treated the patient and is thus not sufficiently objective to offer expert testimony.

C: Yes, because the patient has failed to introduce evidence that the first orthopedist's care fell below the professional standard of care.

A traveler was a passenger on a commercial aircraft owned and operated by an airline. The aircraft crashed into a mountain, killing everyone on board. The flying weather was good. The traveler's legal representative brought a wrongful death action against the airline. At trial, the legal representative offered no expert or other testimony as to the cause of the crash. On the airline's motion to dismiss at the conclusion of the legal representative's case, the court should... A: grant the motion, because the legal representative has offered no evidence as to the cause of the crash. B: grant the motion, because the legal representative has failed to offer evidence negating the possibility that the crash may have been caused by mechanical failure that the airline could not have prevented. C: deny the motion, because the jury may infer that the aircraft crashed due to the airline's negligence. D: deny the motion, because in the circumstances common carriers are strictly liable.

C: deny the motion, because the jury may infer that the aircraft crashed due to the airline's negligence

An eight-year-old child had a habit of riding his bicycle onto a busy highway. His parents knew about this habit but continued to let the child ride his bicycle. One afternoon, the child rode his bicycle down his driveway onto the busy highway and a driver had to stop her car suddenly to avoid colliding with the bike. Because of the sudden stop, the driver's son, who was sitting on the seat without any restraint, was thrown into the dashboard and injured. Had the driver's son been properly restrained in a baby car seat, as required by a state safety statute of which his mother was aware, he would not have been injured. In an action brought on behalf of the driver's son against the child's parents to recover for the son's injuries, the driver's son will.... A: not prevail, because parents are not vicariously liable for the negligent acts of their children. B: not prevail, because the driver's son's injury was attributable to his mother's knowing violation of a safety statute. C: prevail, because the child's parents knew that he sometimes rode into the highway, and they took no steps to prevent it. D: prevail, because the child's riding into the highway was negligent and the proximate cause of the driver's son's injuries.

C: prevail, because the child's parents knew that he sometimes rode into the highway, and they took no steps to prevent it.

A patron ate a spicy dinner at a restaurant on Sunday night. He enjoyed the food and noticed nothing unusual about the dinner. Later that evening, the patron had an upset stomach. He slept well through the night, went to work the next day, and ate three meals. His stomach discomfort persisted, and by Tuesday morning he was too ill to go to work. Eventually, the patron consulted his doctor, who found that the patron was infected with a bacterium that can be contracted from contaminated food. Food can be contaminated when those who prepare it do not adequately wash their hands. The patron sued the restaurant for damages. He introduced testimony from a health department official that various health code violations had been found at the restaurant both before and after the patron's dinner, but that none of the restaurant's employees had signs of bacterial infection when they were tested one month after the incident. The restaurant's best argument in response to the patron's suit would be that... A: no one else who ate at the restaurant on Sunday complained about stomach discomfort. B: the restaurant instructs its employees to wash their hands carefully and is not responsible if any employee fails to follow these instructions. C: the patron has failed to establish that the restaurant's food caused his illness. D: the patron assumed the risk of an upset stomach by choosing to eat spicy food.

C: the patron has failed to establish that the restaurant's food caused his illness

A man was driving his new car along a dark road with the car's high-beam headlights on to illuminate the road. When he saw the headlights of another car appear in the distance, he reached to turn the high beams off. Instead of turning from high-beam to low-beam, the headlights on the car turned off completely. The man tried repeatedly to turn the lights on again but could not do so. He collided with the other car and suffered injuries. The man has brought an action against the manufacturer of the headlight controls in his car, the manufacturer of his car, and the retailer who sold him the car. If the man can establish that a defect in the controls caused the accident, from whom can he recover? A: Only the manufacturer of the car and the retailer of the car. B: Only the manufacturer of the car. C: Only the manufacturer of the headlight controls and the manufacturer of the car. D: All three defendants.

D: All three defendants.

While a plaintiff was leaving an elevator, it suddenly dropped several inches, causing her to fall. An investigation of the accident revealed that the elevator dropped because it had been negligently maintained by an elevator company. The elevator company had a contract with the owner of the building to inspect and maintain the elevator. The plaintiff's fall severely aggravated a preexisting physical disability. If the plaintiff sues the elevator company for damages for her injuries, she should recover... A: nothing, because the elevator company could not reasonably have been expected to foresee the extent of harm that the plaintiff suffered as a result of the accident. B: nothing, because the accident would not have caused significant harm to an ordinarily prudent elevator passenger. C: damages for the full amount of her disability, because a tortfeasor must take its victim as it finds her. D: damages for the injury caused by the falling elevator, including the aggravation of her preexisting disability.

D: Damages for the injury caused by the falling elevator, including the aggravation of her preexisting disability.

The grandson and his friend, both eight years old, were visiting at the grandmother's house when, while exploring the premises, they discovered a hunting rifle in an unlocked gun cabinet. They removed it from the cabinet and were examining it when the rifle, while in the grandson's hands, somehow discharged. The bullet struck and injured the friend. The gun cabinet was normally locked. The grandmother had opened it for dusting several days before the boys' visit, and had then forgotten to relock it. She was not aware that it was unlocked when the boys arrived. At the trial on an action against the grandmother on behalf of the friend, the information above has been admitted into evidence. If the grandmother moves for a directed verdict in her favor at the end of the friend's case, that motion should be... A: granted, because the grandmother is not legally responsible for the acts of her grandson. B: granted, because the grandmother did not recall that the gun cabinet was unlocked. C: denied, because a firearm is an inherently dangerous instrumentality. D: denied, because a jury could find that the grandmother breached a duty of care she owed to the plaintiff.

D: Denied, because a jury could find that the grandmother breached a duty of care she owed to the plaintiff.

Which of the following is an example of making contact with something closely associated with the person of another: A: Hitting a person with a bat. B: Tying a string across a sidewalk to trip a person. C: Poisoning a person's food. D: Grabbing a person's shirt.

D: Grabbing a person's shirt.

While visiting at his son's home, a grandfather tripped on a toy left on the floor by his four-year- old grandson. The grandfather fell and was severely injured. The grandfather regularly visited his son's home and was aware that the grandson routinely left toys scattered about the house. The son had never warned the grandfather to look out for toys. The grandfather brought an action against his son to recover for his injuries, and both the grandfather and the son have moved for directed verdicts as to liability. The jurisdiction has abolished intra-family immunity and applies the traditional rules of landowner liability. What action should the court take? A: Deny both motions and submit the case to the jury based on negligence. B: Deny both motions and submit the case to the jury based on strict liability. C: Grant the grandfather's motion, because the son is liable as a matter of law for failing to warn about the risk of toys being left on the floor. D: Grant the son's motion, because the son had no duty to warn that the grandson might leave toys on the floor.

D: Grant the son's motion, because the son had no duty to warn that the grandson might leave toys on the floor.

A surgeon performed a sterilization operation on a patient. After the surgery, the surgeon performed a test that showed that the patient's fallopian tubes were not severed, as was necessary for sterilization. The surgeon did not reveal the failure of the operation to the patient, who three years later became pregnant and delivered a baby afflicted with a severe birth defect that will require substantial medical care throughout its life. The birth defect resulted from a genetic defect unknown to, and undiscoverable by, the surgeon. The patient brought an action on her own behalf against the surgeon, seeking to recover the cost of her medical care for the delivery of the baby, and the baby's extraordinary future medical expenses for which the patient will be responsible. Which of the following questions is relevant to the lawsuit and currently most difficult to answer? A: Did the surgeon owe a duty of care to the baby with respect to medical services rendered to the patient three years before the baby was conceived? B: Can a person recover damages for a life burdened by a severe birth defect based on a physician's wrongful failure to prevent that person's birth from occurring? C: Did the surgeon owe a duty to the patient to inform her that the sterilization operation had failed? D: Is the patient entitled to recover damages for the baby's extraordinary future medical expenses?

D: Is the patient entitled to recover damages for the baby's extraordinary future medical expenses?

A, intending to throw an empty soda bottle into a trash container, knows that there is a good chance that the bottle will strike B. The bottle strikes B, causing injury. Did A intend that consequence and therefore commit a harmful battery on B? A: Yes, because A knew to a substantial certainty the bottle would contact B. B: Yes, because A's intent to hit the trash container transfers to the resulting contact to B. C: No, because A did not desire to harm. D: No, because A did not know with substantial certainty that the bottle would contact B.

D: No, because A did not know with substantial certainty that the bottle would contact B.

Patient filed a complaint for Intentional Infliction of Emotional Distress alleging Doctor falsely, intentionally and knowingly told Patient he had a terminal illness. Patient also alleged that he suffered depression, headaches, nightmares, and loss of sleep as the result of Doctor's conduct. Has Patient stated a claim on which relief can be granted? A: Yes, because Patient alleged facts sufficient to establish Doctor's conduct is extreme and outrageous. B: Yes, because Patient alleged facts sufficient to establish he suffered severe emotional distress. C: No, because Patient did not allege facts sufficient to establish Doctor's conduct is extreme or outrageous. D: No, because Patient did not allege facts sufficient to establish he suffered severe emotional distress.

D: No, because Patient did not allege facts sufficient to establish he suffered severe emotional distress.

A plaintiff suffered a serious injury while participating in an impromptu basketball game at a public park. The injury occurred when the plaintiff and the defendant, on opposing teams, each tried to obtain possession of the ball when it rebounded from the backboard after a missed shot at the basket. During that encounter, the plaintiff was struck and injured by the defendant's elbow. The plaintiff now seeks compensation from the defendant. At the trial, evidence was introduced tending to prove that the game had been rough from the beginning, that elbows and knees had frequently been used to discourage interference by opposing players, and that the plaintiff had been one of those making liberal use of such tactics. In this action, will the plaintiff prevail? A: Yes, because the defendant intended to strike the plaintiff with his elbow. B: Yes, because the defendant intended to cause harmful or offensive contact with the plaintiff. C: No, because the plaintiff impliedly consented to violent play. D: No, because the defendant did not intentionally use force that exceeded the players' consent.

D: No, because the defendant did not intentionally use force that exceeded the players' consent.

An actress, who played the lead role in a television soap opera, was seriously injured in an automobile accident caused by the defendant's negligent driving. As a consequence of the actress's injury, the television series was canceled, and a supporting actor was laid off. Although the supporting actor looked for other work, he remained unemployed. In an action against the defendant, can the supporting actor recover for his loss of income attributable to the accident? A: Yes, because the defendant's negligence was the cause in fact of the supporting actor's loss. B: Yes, because the supporting actor took reasonable measures to mitigate his loss. C: No, because the defendant had no reason to foresee that by injuring the lead actress he would cause harm to the supporting actor. D: No, because the defendant's liability does not extend to economic loss to the supporting actor that arises solely from physical harm to the lead actress.

D: No, because the defendant's liability does not extend to economic loss to the supporting actor that arises solely from physical harm to the lead actress.

A landlord owns and operates a 12-story apartment building containing 72 apartments, 70 of which are rented. A pedestrian has brought an action against the landlord alleging that while he was walking along a public sidewalk adjacent to the landlord's apartment building a flower pot fell from above and struck him on the shoulder, causing extensive injuries. The action was to recover damages for those injuries. If the pedestrian proves the foregoing facts and offers no other evidence explaining the accident, will his claim survive a motion for directed verdict offered by the defense? A: Yes, because the pedestrian was injured by an artificial condition of the premises while using an adjacent public way. B: Yes, because such an accident does not ordinarily happen in the absence of negligence. C: No, because the landlord is in no better position than the pedestrian to explain the accident. D: No, because there is no basis for a reasonable inference that the landlord was negligent.

D: No, because there is no basis for a reasonable inference that the landlord was negligent

A sporting goods shop was burglarized by an escaped inmate from a nearby prison. The inmate stole a rifle and bullets from a locked cabinet. The burglar alarm at the shop did not go off because the shop's owner had negligently forgotten to activate the alarm's motion detector. Shortly thereafter, the inmate used the rifle ammunition stolen from the shop in a shooting spree that caused injury to several people, including the plaintiff. If the plaintiff sues the shop's owner for the injury she suffered, will the plaintiff prevail? A: Yes, because the plaintiff's injury could have been prevented had the motion detector been activated. B: Yes, because the shop's owner was negligent in failing to activate the motion detector. C: No, because the storage and sale of firearms and ammunition is not an abnormally dangerous activity. D: No, because there is no evidence of circumstances suggesting a high risk of theft and criminal use of firearms stocked by the shop's owner.

D: No, because there is no evidence of circumstances suggesting a high risk of theft and criminal use of firearms stocked by the shop's owner.

As a shopper was leaving a supermarket, an automatic door that should have opened outward opened inward, striking and breaking the shopper's nose. The owner of the building had installed the automatic door. The lease, pursuant to which the supermarket occupied the building, provided that the supermarket was responsible for all maintenance of the premises. The shopper sued the supermarket. At trial, neither the shopper nor the supermarket offered any testimony, expert or otherwise, as to why the door had opened inward. At the close of evidence, both the shopper and the supermarket moved for judgment as a matter of law. How should the trial judge rule? A: Grant judgment for the shopper, because it is undisputed that the door malfunctioned. B: Grant judgment for the supermarket, because the shopper failed to join the owner of the building as a defendant. C: Grant judgment for the supermarket, because the shopper failed to offer proof of the supermarket's negligence. D: Submit the case to the jury, because on these facts negligence may be inferred.

D: Submit the case to the jury, because on these facts negligence may be inferred.

A schizophrenic patient who was institutionalized in a psychiatric facility pushed a nurse down a stairwell at the facility. The nurse, a paid employee of the facility who was trained to care for schizophrenic patients, was injured. The patient is an indigent whose care is paid for by the government. The jurisdiction generally follows the rule that a person with a mental deficiency is held to the standard of a reasonable person. In a negligence action brought by the nurse against the patient, the patient's lawyer will argue that the patient should not be held responsible for the nurse's injury. Which of the following facts will be LEAST helpful to the patient's lawyer's argument? A: The nurse was a professional caregiver. B: The nurse was trained to care for patients with schizophrenia. C: At the time she pushed the nurse, the patient thought she was being attacked by an elephant. D: The patient is an indigent whose care is paid for by the government.

D: The patient is an indigent whose care is paid for by the government.

A pedestrian was crossing a street in a crosswalk when a woman walking just ahead of him was hit by a truck. The pedestrian, who had jumped out of the way of the truck, administered CPR to the woman, who was a stranger. The woman bled profusely, and the pedestrian was covered in blood. The woman died in the ambulance on the way to the hospital. The pedestrian became very depressed immediately after the incident and developed physical symptoms as a result of his emotional distress. The pedestrian has brought an action against the driver of the truck for negligent infliction of emotional distress. In her defense, the driver asserts that she should not be held liable, because the pedestrian's emotional distress and resulting physical symptoms are not compensable. What is the strongest argument that the pedestrian can make in response to the driver's defense? A: The pedestrian saw the driver hit the woman. B: The pedestrian was acting as a Good Samaritan. C: The pedestrian was covered in the woman's blood and developed physical symptoms as a result of his emotional distress. D: The pedestrian was in the zone of danger.

D: The pedestrian was in the zone of danger

A graduate student who was moving needed cardboard boxes, so she went to her usual grocery store to look for some. The store regularly gave repeat customers empty boxes to promote goodwill. Seeing no boxes outside, the student entered the store and asked a store employee for help. The employee pointed toward the rear of the store and said that all the empty boxes were in the storeroom. The student went into the storeroom through a door with a sign that said: "Keep Out. Employees Only." While in the storeroom, she tripped on a fold in a floor mat and fell into a stack of wooden crates. The topmost crate fell on the student, causing a head injury. The student has sued the store to recover for her injury. Which statement below is the most appropriate characterization of the student and her conduct under traditional common law rules? A: The student assumed the risk, because she knew that the storeroom was not normally accessible to the public. B: The student was a licensee in the store, because she had no intention of making a purchase at the store during the box-hunting visit. C: The student was a trespasser in the storeroom, because she ignored the sign on the door barring entrance to the storeroom. D: The student was an invitee in the storeroom, because she had permission to enter the storeroom consistent with the store's policy of making its empty boxes available to repeat customers.

D: The student was an invitee in the storeroom, because she had permission to enter the storeroom consistent with the store's policy of making its empty boxes available to repeat customers.

Player A used his hockey stick to intentionally strike Player B with a two-handed slash to the neck from behind that caused Player B to lose his helmet. This contact is prohibited by the rules of hockey because of the high risk of severe injury. Left Wing collapsed when he suffered a seizure on the ice and a severe concussion. Which of the following is MOST likely to impact a court's decision about whether Player B consented to the contact? A: Both players were professionals. B: The players had signed waivers of liability relative to the inherent risks of hockey. C: The injury suffered was very serious and could impact Player B's ability to continue to play hockey. D: This type of slash is prohibited by the rules of hockey because of the high risk of severe injury.

D: This type of slash is prohibited by the rules of hockey because of the high risk of severe injury.

An ordinance in a small town required all restaurants to designate smoking and nonsmoking sections for their customers. A cigarette smoker and a nonsmoker were seated at adjoining tables in a small restaurant. The smoker's table was in the smoking section, and the nonsmoker's table was in the nonsmoking section. When the smoker lit a cigarette, the nonsmoker politely requested that he not smoke, explaining that she had a severe allergy to cigarette smoke. The smoker ignored the nonsmoker's request and continued to smoke. As a result, the nonsmoker was hospitalized with a severe allergic reaction to the smoke. The nonsmoker brought a battery action against the smoker.Which of the following questions will NOT be an issue in the battery action? A: Did the smoker intend to cause the nonsmoker's contact with the cigarette smoke? B: Does smoke have the physical properties necessary for making the kind of contact required for battery? C: Is contact with cigarette smoke from a lawful smoking section in a restaurant the kind of contact one must endure as a voluntary restaurant patron? D: Was the smoker's conduct unreasonable under the circumstances?

D: Was the smoker's conduct unreasonable under the circumstances?

A rancher and his neighbor were involved in a boundary dispute. In order to resolve their differences, each drove his truck to an open pasture area on his land where the two properties were separated by a fence. The rancher was accompanied by four friends, and the neighbor was alone. The neighbor got out of his truck and walked toward the fence. The rancher got out but simply stood by his truck. When the neighbor came over the fence, the rancher shot him, inflicting serious injury. In a battery action brought by the neighbor against the rancher, the rancher testified that he actually thought his neighbor was armed, although he could point to nothing that would have reasonably justified this belief. Is the neighbor likely to prevail? A: No, because the rancher was standing on his own property and had no obligation to retreat. B: No, because the rancher suspected that the neighbor was armed. C: Yes, because deadly force is never appropriate in a property dispute. D: Yes, because it was unreasonable for the rancher to consider the use of a gun necessary for self-defense.

D: Yes, because it was unreasonable for the rancher to consider the use of a gun necessary for self-defense.

When a tire of a motorist's car suffered a blowout, the car rolled over and the motorist was badly injured. Vehicles made by the manufacturer of the motorist's car have been found to be negligently designed, making them dangerously prone to rolling over when they suffer blowouts. A truck driver who was driving behind the motorist when the accident occurred stopped to help. Rescue vehicles promptly arrived, and the truck driver walked along the side of the road to return to his truck. As he approached his truck, he was struck and injured by a speeding car. The truck driver has sued the manufacturer of the injured motorist's car. Is the truck driver likely to prevail in a suit against the car manufacturer? A: No, because the car manufacturer's negligence was not the proximate cause of the truck driver's injuries. B: No, because the truck driver assumed the risk of injury when he undertook to help the motorist. C: Yes, because it is foreseeable that injuries can result from rollovers. D: Yes, because the car manufacturer's negligence caused the dangerous situation that invited the rescue by the truck driver.

D: Yes, because the car manufacturer's negligence caused the dangerous situation that invited the rescue by the truck driver.

An eight-year-old child went to the grocery store with her mother. The child pushed the grocery cart while her mother put items into it. The child's mother remained near the child at all times. Another customer in the store noticed the child pushing the cart in a manner that caused the customer no concern. A short time later, the cart the child was pushing struck the customer in the knee, inflicting serious injury. Assume that the child was negligent and the child's mother did not adequately supervise the child. If the customer brings an action, based on negligence, against the child's mother, will the customer prevail? A: Yes, because the child was negligent. B: Yes, because the child's mother is responsible for any harm caused by the child. C: Yes, because the child's mother assumed the risk of her child's actions. D: Yes, because the child's mother did not adequately supervise the child's actions.

D: Yes, because the child's mother did not adequately supervise the child's actions.

A plaintiff entered a drug store to make some purchases. As he was searching the aisles for various items, he noticed a display card containing automatic pencils. The display card was on a high shelf behind a cashier's counter. The plaintiff saw a sign on the counter that read, "No Admittance, Employees Only." Seeing no clerks in the vicinity to help him, the plaintiff went behind the counter to get a pencil. A clerk then appeared behind the counter and asked whether she could help him. He said he just wanted a pencil and that he could reach the display card himself. The clerk said nothing further. While reaching for the display card, the plaintiff stepped sideways into an open shaft and fell to the basement, 10 feet below. The clerk knew of the presence of the open shaft, and had reason to believe that the plaintiff had not noticed it when stepping behind the counter. The plaintiff sued the drug store to recover damages for the injuries he sustained in the fall. The jurisdiction has adopted a rule of pure comparative negligence, and it follows traditional common-law rules governing the duties of a land possessor. Will the plaintiff recover a judgment against the drug store? A: No, because the plaintiff was a trespasser. B: No, because the plaintiff's injuries did not result from the defendant's willful or wanton misconduct. C: Yes, because the premises were defective with respect to a public invitee. D: Yes, because the clerk had reason to believe that the plaintiff was unaware of the open shaft.

D: Yes, because the clerk had reason to believe that the plaintiff was unaware of the open shaft.

A driver was injured when one of the tires on her car ruptured, causing the car to veer off the road. The rupture resulted from a defect in the tire that existed at the time it was sold by the manufacturer to a retailer, who sold it to the driver and installed it on her car. The driver sued the manufacturer on a strict products liability theory. During discovery, the driver's attorney obtained a memorandum that had circulated among engineers employed by the manufacturer. The memorandum acknowledged that undetectable defects would cause ruptures in approximately one out of every 10,000 tires sold. The manufacturer sells approximately 10 million tires per year. After discovering this memorandum, the driver's attorney filed a timely amended complaint, adding a claim for battery. The manufacturer has moved for partial summary judgment, seeking dismissal of the battery claim, based on the undisputed facts set out above. If no additional facts are provided to the court concerning the risk of tire rupture, should the court grant the motion? A: No, because the evidence would permit a reasonable jury to conclude that a responsible manufacturer would have taken greater care to prevent ruptures. B: No, because the evidence would permit a reasonable jury to conclude that the manufacturer knew to a substantial certainty that consumers would be injured by tire ruptures. C: Yes, because a battery claim is inconsistent with a claim for strict products liability. D: Yes, because the driver cannot establish that the manufacturer intended or knew to a substantial certainty that its placement of the tire into the stream of commerce would cause her to suffer a harmful or offensive contact.

D: Yes, because the driver cannot establish that the manufacturer intended or knew to a substantial certainty that its placement of the tire into the stream of commerce would cause her to suffer a harmful or offensive contact.

A shopper slipped and fell in a grocery store, injuring her wrist. In a medical malpractice action against the doctor who treated her, the shopper alleges that the doctor worsened the injury by his treatment. Normally, competent medical treatment would have resulted in a complete cure of the wrist injury. The doctor is seeking to implead the grocery store. The grocery store contends that its alleged negligence was not a proximate cause of any of the injuries allegedly caused by the doctor. Should the court allow the doctor to implead the grocery store? A: No, because medical malpractice and the negligence of a possessor of property involve different standards of care. B: No, because the alleged negligent acts of the grocery store and the doctor occurred sequentially and not concurrently. C: No, because the fact-finder could find that the shopper sustained a single indivisible injury proximately caused by the negligence of both the grocery store and the doctor. D: Yes, because the fact-finder could assign some of the responsibility for the shopper's injuries to the grocery store

D: Yes, because the fact-finder could assign some of the responsibility for the shopper's injuries to the grocery store.

A 13-year-old girl was operating a high-speed motorboat. The boat was towing a 9-year-old boy in an inner tube tied to the rear of the motorboat by a rope. The rope became tangled around the boy's foot, causing him to suffer severe injuries. In a suit brought on the boy's behalf against the girl, the boy has introduced uncontroverted evidence that the girl drove carelessly in such a way as to entangle the boy in the rope. Is the boy likely to prevail? A: No, because the boy assumed the risk. B: No, because the girl was too young to be expected to appreciate and avoid the risk she exposed the boy to. C: Yes, because children of the girl's age should have the capacity to operate motorboats. D: Yes, because the girl will be held to an adult standard of care.

D: Yes, because the girl will be held to an adult standard of care.

A male employee who worked at a psychiatric hospital had sexual relations with a patient in her room at the hospital. The patient was severely mentally disabled, and although the patient did not initiate the encounter, she did not protest. A tort action was brought on the patient's behalf against the hospital. The hospital contends that the employee's actions were outside the scope of his employment. Is the patient likely to prevail? A: No, because the employee's actions were outside the scope of his employment. B: No, because the patient encouraged the employee's actions by not protesting. C: Yes, because the employee worked for the hospital. D: Yes, because the hospital failed to use reasonable care to protect the patient from such conduct.

D: Yes, because the hospital failed to use reasonable care to protect the patient from such conduct.

A trucker driving down an isolated country road late one night struck cattle that had escaped from a farmer's pen and wandered into the road. The trucker was unable to stop before hitting the cattle but was not driving carelessly. While he was not injured in the collision, the trucker sustained damage to his truck and lost income during the time it took to repair the truck. The trucker sued the farmer for his damages and invoked the doctrine of res ipsa loquitur. At trial, the farmer introduced evidence that his cattle pen was of a sufficient height to prevent cattle from stepping over it and was constructed of thick steel pipe sitting in concrete with a substantial top rail. A sturdy pen such as this one would be more difficult for cattle to break through than one constructed of barbed wire or electric wire. Should the trial court allow the case to go to the jury with a res ipsa loquitur instruction? A: No, because it is possible that a third party wrongfully let the cattle out of the pen. B: No, because the trucker must submit direct evidence of negligence in order to invoke the res ipsa loquitur doctrine. C: Yes, because the farmer is strictly liable for harm caused by his escaping cattle. D: Yes, because the jury could conclude that cattle would not ordinarily escape a strong, secure cattle pen in the absence of negligence.

D: Yes, because the jury could conclude that cattle would not ordinarily escape a strong, secure cattle pen in the absence of negligence.

The owner of a shopping mall hired a construction company to design and construct a new entryway to the mall. The construction company negligently selected an unusually slippery material for the floor covering. A week after the entryway was completed, a customer who had come to the mall to buy cosmetics slipped on the floor of the entryway, sustaining injuries. The customer sued the mall owner for the construction company's negligent design of the mall's entryway. Will the injured customer be likely to recover damages? A: No, because the construction company will likely be considered an independent contractor. B: No, because no other customers had previously slipped on the floor. C: Yes, because the customer intended to make a purchase at the mall. D: Yes, because the mall's duty to maintain safe conditions was nondelegable.

D: Yes, because the mall's duty to maintain safe conditions was nondelegable.

A man's car sustained moderate damage in a collision with a car driven by a woman. The accident was caused solely by the woman's negligence. The man's car was still drivable after the accident. Examining the car the next morning, the man could see that a rear fender had to be replaced. He also noticed that gasoline had dripped onto the garage floor. The collision had caused a small leak in the gasoline tank. The man then took the car to a mechanic, who owns and operates a body shop, and arranged with the mechanic to repair the damage. During their discussion the man neglected to mention the gasoline leakage. Thereafter, while the mechanic was loosening some of the damaged material with a hammer, he caused a spark, igniting vapor and gasoline that had leaked from the fuel tank. The mechanic was severely burned. The mechanic has brought an action to recover damages against the man and woman. The jurisdiction has adopted a pure comparative negligence rule in place of the traditional common law rule of contributory negligence. In this action, will the mechanic obtain a judgment against the woman? A: No, because there is no evidence that the woman was aware of the gasoline leak. B: No, because the mechanic would not have been harmed had the man warned him about the gasoline tank. C: Yes, because the mechanic was not negligent in failing to discover the gasoline leak himself. D: Yes, because the mechanic's injury was a proximate consequence of the woman's negligent driving.

D: Yes, because the mechanic's injury was a proximate consequence of the woman's negligent driving.

A pedestrian was injured when hit by a chair that was thrown from an upper-story hotel window. The pedestrian sued the occupants of all the rooms from which the chair might have been thrown. At trial, the pedestrian has been unable to offer any evidence as to the exact room from which the chair was thrown. The defendants have filed a motion for a directed verdict. Should the court grant the motion? A: No, because it is unreasonable to expect the pedestrian to prove which of the defendants caused the harm. B: No, because of the doctrine of alternative liability. C: Yes, because a plaintiff always has the burden to prove that a particular defendant's conduct was the factual cause of the plaintiff's physical harm. D: Yes, because the pedestrian has failed to offer evidence that the defendants jointly engaged in tortious conduct.

D: Yes, because the pedestrian has failed to offer evidence that the defendants jointly engaged in tortious conduct.

A homeowner owned a large poisonous snake which had been defanged and was kept in a cage. A storm damaged the homeowner's house and the snake's cage, allowing the snake to escape. During the cleanup after the storm, a volunteer worker came across the snake. The worker tried to run away from the snake and fell, breaking his arm. In a suit by the worker against the homeowner based on strict liability in tort to recover for his injury, will the worker prevail? A: No, because the snake's escape was caused by a force of nature. B: No, because the worker should have anticipated an injury during his volunteer work. C: Yes, because the homeowner did not take adequate precautions to secure the snake. D: Yes, because the worker's injury was the result of his fear of the escaped snake.

D: Yes, because the worker's injury was the result of his fear of the escaped snake.

A fire that started in the defendant's warehouse spread to the plaintiff's adjacent warehouse. The defendant did not intentionally start the fire, and the plaintiff can produce no evidence as to how the fire started. However, the defendant had failed to install a sprinkler system, which was required by a criminal statute. The plaintiff can produce evidence that had the sprinkler system been installed, it could have extinguished the fire before it spread. In an action by the plaintiff against the defendant to recover for the fire damage, is it possible for the plaintiff to prevail? A: No, because the statute provides only for criminal penalties. B: No, because there is no evidence that the defendant negligently caused the fire to start. C: Yes, because a landowner is strictly liable for harm to others caused by the spread of fire from his premises under the doctrine of Rylands v. Fletcher. D: Yes, because the plaintiff was harmed as a result of the defendant's violation of a statute that was meant to protect against this type of occurrence.

D: Yes, because the plaintiff was harmed as a result of the defendant's violation of a statute that was meant to protect against this type of occurrence.

The plaintiff is being treated by a physician for asbestosis, an abnormal chest condition that was caused by his on-thejob handling of materials containing asbestos. His physician has told him that the asbestosis is not presently cancerous, but that it considerably increases the risk that he will ultimately develop lung cancer. The plaintiff brought an action for damages, based on strict product liability, against the supplier of the materials that contained asbestos. The court in this jurisdiction has ruled against recovery of damages for negligently inflicted emotional distress in the absence of physical harm. If the supplier is subject to liability to the plaintiff for damages, should the award include damage for emotional distress he has suffered arising from his knowledge of the increased risk that he will develop lung cancer? A: No, because the plaintiff's emotional distress did not cause his physical condition. B: No, because the court does not recognize a cause of action for an increased risk of cancer. C: Yes, because the supplier of a dangerous product is strictly liable for the harm it causes. D: Yes, because the plaintiff's emotional distress arises from bodily harm caused by his exposure to asbestos.

D: Yes, because the plaintiff's emotional distress arises from bodily harm caused by his exposure to asbestos.

A chemical company's plant was located in a residential community. The manufacturing process used at the plant generated a toxic chemical as a by- product. The chemical was stored in a state-of-the- art tank on the site before being moved to an off-site disposal facility. The on-site storage arrangement conformed to the requirements of reasonable care and to the applicable government regulations. However, the storage of the toxic chemical created a foreseeable and highly significant risk of physical harm even when reasonable care was exercised. Despite the chemical company's proper use and care of the storage tank, toxic fumes escaped from the tank and made residents of the area seriously ill. No state or federal statutes address the issue of the company's liability. In an action by one of the affected residents against the chemical company, will the resident be likely to prevail? A: No, because the chemical company conformed to the requirements of reasonable care and to the applicable government regulations. B: No, because the chemical company used a state-of-the-art storage tank. C: Yes, because the chemical company is strictly liable in tort for any harm caused by the toxic chemicals it produced. D: Yes, because the storage of toxic chemicals in a residential community created a highly significant risk of physical harm even when reasonable care was exercised.

D: Yes, because the storage of toxic chemicals in a residential community created a highly significant risk of physical harm even when reasonable care was exercised.

Patient is a resident at a facility for moderately emotionally disturbed adults. As part of his treatment, he is encouraged to engage in outdoor activities. Patient was playing a game with Nurse which involved tossing bean bags at a target consisting of an inclined wooden platform with a hole at one end. Suddenly, Patient turned menacingly towards Nurse and threw the bean bag, with great force, directly at her. The bean bag hit Nurse in the head, causing her to have a severe bruise. If Nurse sues Patient for battery in a jurisdiction adopting a dual intent rule, will she prevail? A: No, because a mentally ill adult is deemed by law incapable of forming the intent to injure another. B: No, because Nurse impliedly consented to the risks of the game when she agreed to play handball. C: Yes, because a mentally ill adult is expected to behave as a reasonable person under the circumstances. D: Yes, unless the mental illness prevents Patient from having the ability to form the intent to harm.

D: Yes, unless the mental illness prevents Patient from having the ability to form the intent to harm.

Patient is a resident at a facility for moderately emotionally disturbed adults. As part of his treatment, he is10 encouraged to engage in outdoor activities. Patient was playing a game with Nurse which involved tossing bean bags at a target consisting of an inclined wooden platform with a hole at one end. Suddenly, Patient turned menacingly towards Nurse and threw the bean bag, with great force, directly at her. The bean bag hit Nurse in the head, causing her to have a severe bruise. If Nurse sues Patient for battery in a jurisdiction adopting a single intent rule, will she prevail? A: No, because a mentally ill adult is deemed by law incapable of forming the intent to injure another. B: No, because Nurse impliedly consented to the risks of the game when she agreed to play the game. C: Yes, because a mentally ill adult is expected to behave as a reasonable person under the circumstances. D: Yes, unless the mental illness prevents Patient from having the ability to form the intent to touch.

D: Yes, unless the mental illness prevents Patient from having the ability to form the intent to touch.

While a woman was in her kitchen, she heard the screech of automobile tires. She ran to the window and saw a tricycle flying through the air. The tricycle had been hit by a car driven by a young man, who had been speeding. She also saw a child's body in the grass adjacent to the street. As a result of her shock from this experience, the woman suffered a heart attack. In a claim by the woman against the young man, the woman's right to recovery will depend on whether... A: a person can recover damages based on the defendant's breach of a duty owed to another. B: it is foreseeable that a person may suffer physical harm caused solely by an injury inflicted on another. C: a person can recover damages caused by shock unaccompanied by bodily impact. D: a person can recover damages for harm resulting from shock caused solely by another's peril or injury. (D)

D: a person can recover damages for harm resulting from shock caused solely by another's peril or injury.

An eight-year-old child went to the grocery store with her mother. The child pushed the grocery cart while her mother put items into it. The child's mother remained near the child at all times. Another customer in the store noticed the child pushing the cart in a manner that caused the customer no concern. A short time later, the cart the child was pushing struck the customer in the knee, inflicting serious injury. If the customer brings an action, based on negligence, against the grocery store, the store's best defense will be that... A: a store owes no duty to its customers to control the use of its shopping carts. B: a store owes no duty to its customers to control the conduct of other customers. C: any negligence of the store was not the proximate cause of the customer's injury. D: a supervised child pushing a cart does not pose an unreasonable risk to other customers.

D: a supervised child pushing a cart does not pose an unreasonable risk to other customers.

A plaintiff suffered a severe loss when his manufacturing plant, located in a shallow ravine, was flooded during a sustained rainfall. The flooding occurred because the city had failed to maintain its storm drain, which was located on city land above the plaintiff's premises, and because a railroad had failed to maintain its storm drain, which was located on railroad land below the plaintiff's premises. The flooding would not have occurred if either one of the two storm drains had been maintained properly. The plaintiff sued the railroad to recover compensation for his loss. The evidence in the case established that the failures of the two drains were caused by the respective negligence of the city and the railroad. There is no special rule insulating the city from liability. In his action against the railroad, the plaintiff should recover... A: nothing, because he should have joined the city, without whose negligence he would have suffered no loss. B: nothing, because he did not introduce evidence that enables the court reasonably to apportion responsibility between the city and the railroad. C: one-half his loss, in the absence of evidence that enables the court to allocate responsibility fairly between the city and the railroad. D: all of his loss, because but for the railroad's negligence none of the flooding would have occurred

D: all of his loss, because but for the railroad's negligence none of the flooding would have occurred.

In an action brought against a defendant by a pedestrian's legal representative, the only proof that the legal representative offered on liability were that: (1) the pedestrian was killed instantly while walking on the shoulder of the highway; (2) the defendant was driving the car that struck the pedestrian; and (3) there were no living witnesses to the accident other than the defendant, who denied negligence. The jurisdiction has adopted a rule of pure comparative negligence.If, at the end of the plaintiff's case, the defendant moves for directed verdict, the trial judge should... A: grant the motion, because the legal representative has offered no specific evidence from which reasonable jurors may conclude that the defendant was negligent. B: grant the motion, because it is just as likely that the pedestrian was negligent as that the defendant was negligent. C: deny the motion, because the pedestrian was in violation of the state highway code. D: deny the motion, because, in the circumstances, negligence on the part of the defendant may be inferred.

D: deny the motion, because, in the circumstances, negligence on the part of the defendant may be inferred.

A plaintiff and a man were passengers sitting in adjoining seats on a flight on an airline. There were many empty seats on the aircraft. During the flight, a flight attendant served the man nine drinks. As the man became more and more obviously intoxicated and attempted to engage the plaintiff in a conversation, the plaintiff chose to ignore the man instead of changing seats. This angered the man, who suddenly struck the plaintiff in the face, giving him a black eye. The flight attendant had witnessed the man becoming violent but chose to not get involved. If the plaintiff asserts a claim for damages against the airline based on negligence, the plaintiff will.... A: not recover, because a person is not required by law to come to the assistance of another who is imperiled by a third party. B: not recover, because the plaintiff could easily have moved to another seat. C: recover, because a common carrier is strictly liable for injuries suffered by a passenger while aboard the carrier. D: recover, because the flight attendants should have perceived the man's condition and acted to protect the plaintiff before the blow was struck.

D: recover, because the flight attendants should have perceived the man's condition and acted to protect the plaintiff before the blow was struck.


Kaugnay na mga set ng pag-aaral

DECA Finance Cluster Exam Practice Info

View Set

Paramedic National Test Prep: Obstetrics & Pediatrics

View Set

Simulation Lab 10.2: Module 10 Install Linux in VM

View Set

HIM 3200 Final Study, HIM 3200 Epidemiology & Biostatistics Midterm, HIM 3200 Epidemiology & Biostatistics Final, HIM 3200 Epidemiology & Biostatistics Midterm, HIM 3200- Midterm, Quiz Questions, HIM 3200 Final Study

View Set

SSUSH 17 Analyze the causes and consequences of the Great Depression.

View Set